Vous êtes sur la page 1sur 90

Question 1

A 34year-old male is brought to ER after a motor vehicle accident Examination showed a large
bruise over his anterior chest wall. Several days later, the bruise has a greenish color that is best
explained by the action of:

Answers
A. Ferrochelatase
B. Uroporphyrinogen decarboxylase
C. Heme oxygenase
D. Bilirubin glucuronyl transferase
E. Methemoglobin reductase

Explanation
Normally, after red cells have reached the end of their life span, they are phagocytosed and
degraded by cells of the reticuloendothelial system. These cells cleave globin derived from
hemoglobin molecule into its constituent amino acids. Released iron is returned to body's iron
stores. Heme moiety of hemoglobin molecule is cleaved to produce carbon dioxide and biliverdin.
Biliverdin is reduced to bilirubin and subsequently transported to the liver bound to albumin.
The patient described in the question above is having a resolving hematoma produced due to
multiple injuries. In this case - as happens normally- heme oxygenase converts heme derived from
RBCs to biliverdin. Biliverdin is a green pigment and is responsible for the green color that
develops in ecchymoses and hematomas several days after an injury.

Choice A: Ferrochelatase is the final enzyme in the heme synthetic pathway. Ferrochelatase is
involved in heme anabolism, rather than heme anabolism.

Choice B: Uroporphyrinogen decarboxylase like ferrochelatase, is also involved in the production of


heme and not in its degradation. Urooporphyrinogen decarboxylase enzyme is defective in
porphyria cutanea tarda (PCT), the most common form of porphyria

Choice D: Bilirubin glucuronyl transferase is the enzyme necessary for the conjugation of bilirubin
to glucuronic acid. Either, lack of this enzyme, or presence of drugs that interfere with glucuronyl
transferase impair the ability of liver to conjugate bilirubin.

Choice E: Methemeglobin reductase is an enzyme present in erythrocytes that catalyzes the


reduction of methemoglobin to hemoglobin via oxidation of NADH. Physiological significance of
this enzyme is uncertain. The enzyme can be activated if methylene blue or other artificial electron
acceptors are introduced into body’s milieu. No endogenous intermediate electron carrier has been
found.

Educational Objective:
Heme oxygenase enzyme converts heme to biliverdin, a pigment that causes green color to develop
in hematomas and ecchymoses, 4 to 5 days after an injury. The process of biliverdin release is
similar to what happens during physiologic catabolism of hemoglobin derived from senescent
erythrocytes.
Question 2
A 21-year-old laboratory worker experiences rapid-onset breathing difficulty, palpitations and
flushing of skin. He has no significant past medical history. He takes loratadine occasionally for
seasonal allergies. The patient is suspected to have accidental poisoning, for which he is
administered amyl nitrite immediately, via inhalation, from a laboratory safely kit. Amyl nitrite
affects the affinity of hemoglobin for which of the following moieties?

Answers
A. Carbon dioxide
B. 2.3-biphosphoglycerate
C. Carbon monoxide
D. Cyanide
E. Iron
F. Lead

Explanation:
Cyanide binds to a variety of iron-containing enzymes, the most important of them being
cytochrome oxidase complex. This complex is critical in metabolism for electron transport during
oxidative phosphorylation. Binding of minute amounts of cyanide to this complex can inhibit
aerobic metabolism and result in rapid death of the victim.
The typical clinical syndrome produced in cyanide poisoning is - rapidly-developing cutaneous
flushing, tachypnea, headache and tachycardia. This is often accompanied by nausea, vomiting,
confusion and weakness Respiratory distress and cardiac dysfunction may follow. Laboratory
studies will indicate severe lactic acidosis in conjunction with a diminished difference between
arterial and venous oxygen content (i.e. the venous blood is still highly oxygenated).

Drugs (Nitrites)

The antidotal effect of nitrites for cyanide poisoning has been recognized since the late nineteenth
century. Nitrites act primarily in cyanide poisoning by inducing formation of methemoglobin.
Methemoglobin cannot carry oxygen, but it does have a high affinity for cyanide. Thus
administration of amyl nitrite soon after cyanide exposure will sequester cyanide in blood and
regenerate active cytochrome. This will keep the cyanide away from mitochondria and other sites
where this compound exerts its toxic effects. Sodium thiosulfate is also used for cyanide poisoning.
It combines with cyanide to form less toxic thiocyanate, which is excreted in the urine.
Choices A, B and C: Methemoglobin, which is formed when amyl nitrite is administered does not
have high affinity for molecules like carbon monoxide, carbon dioxide and 2,3 biphosphoglycerate

Choice E: The affinity of hemoglobin for iron is not affected by nitrite administration, although,
nitrites do oxidize heme iron to its ferric (III) state.

Choice F: Lead poisoning causes defective heme synthesis. Lead poisoning is treated first and
foremost by the avoidance of lead ingestion. If actually ingested, chelation therapy such as with
dimercaprol or Ca Na EDTA, should be initiated

Educational Objective:
Nitrites are oxidizing agents that are effective in treatment of cyanide poisoning. This is due to their
ability to produce methemoglobinemia. Methemoglobin contains ferric rather than ferrous iron.
Cyanide binds to ferric iron more avidly than to mitochondrial cytochrome oxidase complex and
thus nitrites save vital mitochondrial enzymes from toxic effect of cyanide.
Question 3
Circulating erythrocytes often contain enzymes that are known to participate in heme synthesis.
Despite this they are unable to synthesize heme because they lack which of the following cellular
organelles?

A. Nucleus
B. Mitochondria
C. Endoplasmic reticulum
D. Golgi apparatus
E. Proteasomes
F. Peroxisomes

Explanation
Heme is synthesized in virtually every organ, but the principal sites of heme synthesis are:
erythrocytes and hepatocytes. Erythrocytes utilize the iron-containing heme to form hemoglobin,
the major oxygen-carrying molecule the body. Hepatocytes use heme primarily in microsomal
cytochrome p450 system.
Maturing erythrocytes lose their ability to synthesize heme when they lose their mitochondria.
Mitochondria are necessary for the first and final three steps of home synthesis. During
erythropoeisis, before leaving the cell cycle to form mature red blood cells, developing erythrocyte
precursors divide many times, and finally lose their nuclei and mitochondria. Heme synthesis occurs
partly in mitochondria and partly in the cytoplasm of developing erythrocytes. As mature
erythrocytes do not have mitochondria, they lose the ability to generate heme and thus hemoglobin.
An illustration of the cellular location and the corresponding enzymes involved in the heme
synthetic pathway is shown above.

Choice B: Mitochondria are the location for heme biosynthesis. These organelles are absent in
mature erythrocytes.
Choice A, C, D, E and F: Although nucleus is also not present in mature erythrocytes, it is not
involved in heme biosynthetic pathway. Other organelles mentioned above too are not involved in
heme synthesis

Educational objective:
Maturing erythrocytes lose their ability to synthesize heme when they lose their mitochondria.
Mitochondria are essential for the first and final three steps of heme synthesis.
Question 4
An infant born to a Greek immigrant appears healthy at birth but subsequently develops transfusion-
dependent hemolytic anemia by the age of 6 months. His erythrocytes contain insoluble aggregates
of hemoglobin subunits. The child developed normally in-utero, because during that period he was
produceing high quantities of:

Answers

Explanation:

In adults, hemoglobin A is the predominant form of hemoglobin (HbA). It is a tetramer consisting of


two alpha and two beta chains. Normally, synthesis of alpha and beta chains is tightly regulated.
Hemoglobin formation begins within a few weeks of conception. The initial hemoglobin formed in
fetus, in-utero, is called embryonic hemoglobin (hemoglobin Gower). This hemoglobin is composed
of two zeta (ζ) and two epsilon {ε) chains (ζ 2ε2) and is produced in the embryonic yolk sac. Within a
few weeks, the fetal liver starts synthesizing hemoglobin F (fetal hemoglobin). This form of
hemoglobin is composed of two alpha and two gamma chains. HbF is the major hemoglobin in
fetus during last few months of gestation, and in infants during first few weeks of postnatal life.
HbA synthesis starts during the final month of gestation and it gradually replaces HbF during the
postnatal life. Knowing the chronology of fetal hemoglobin formation and the gradual transition to
adult hemoglobin (HbA) formation is important in understanding the relationship between clinical
manifestations and postnatal age in beta -thalassemia.
Thalassemias are hereditary hemolytic anemias resulting from defective synthesis of globin chains.
As described above, synthesis of alpha and beta globin chains is tightly regulated. In patients with
thalassemia, synthesis of either alpha or beta chains is defective. Beta-thalassemia is caused by
defective synthesis of beta chains. There are two copies of the beta globin gene (one from each
parent). If only one gene is defective, the patient will have beta thalassemia trait (beta thalassemia
minor). Such an individual will not have significant anemia. Defect of both beta globin genes results
in severe hemolytic anemia known as beta-thalassemia major. In this disease, alpha chains are
produced normally but they cannot form stable hemoglobin tetramers due to the lack of beta globin
chains. Failure to form stable hemoglobin leads to precipitation of alpha chains and premature lysis
of red blood cells. Beta thalassemia will not become symptomatic as long as hemoglobin with
significant amounts of gamma chains is present, because gamma chains would make up for the
absence of beta chains required for formation of HbA tetramers. Thus, in late gestation and during
early postnatal life, expression of deficient beta chain formation is offset by gamma chain
production.

Correct answer is choice C: In-utero and in early post-natal life, production of gamma chains
compensates for defective HbA synthesis. As gamma chain production wanes during infancy,
patients develop anemia and become symptomatic.
Choice A: Alpha globin chain is a normal component of both HbA and HbF. Alpha globin chain is
able to combine with gamma globin chains to form HbF. This allows patients of beta thalassemia
major to remain asymptomatic in-utero and during first few months following birth.

Choice B: beta globin chain synthesis is defective in patients with beta-thalassemia.

Choice D: delta globin is a minor globin gene that is expressed at very low levels in normal adults.
Two delta globin chains and two alpha globin chains combine to form hemoglobin A2.

Choice E: zeta globin is a component of hemoglobin Gower, the initial hemoglobin formed by the
embryo during very early embryogenesis

Educational Objective:
HbF contains gamma globin chains instead of beta globin chains. Patients with homozygotic beta-
thalassemia ((ß-thalassemia major) are asymptomatic at birth due to production of gamma globin
chains that form fetal hemoglobin. In such patients, during early post-natal life, change in the type
of hemoglobin produced i.e. from fetal (HbF) to the adult (HbA) form is responsible for anemia and
development of symptoms.
Question 5
It is seen that in healthy volunteers, the chloride content of erythrocytes is much lower in arterial
blood than in venous blood. The action of which of the following is mostly responsible for this
observed difference?

Answers
A. Spectrin
B. Na/K ATPase
C. Carbonic anhydrase
D. 2,3-biphosphoglycerate mutase
E. Glucoso-6-phosphate dehydrogenase

Explanation

Hemoglobin is found only in red blood cells (RBCs) and is responsible not only for oxygen delivery
to the tissues, but also for carrying carbon dioxide from tissues to the lungs. Hemoglobin carries
carbon dioxide as carbamino compounds formed by reaction of CO 2 with amino groups of proteins.
The reaction can be represented as follows:

Out of 50 ml of CO2 in each 100 ml of arterial blood, 3 ml is carried as carbamino compounds and
44 ml as HCO3-. Carbon dioxide produced by tissue respiration enters red blood cells and is
hydrated by carbonic anhydrase to form carbonic acid. Carbonic acid dissociates into HCO 3- and H+.
Since the rise in HCO3- in RBCs is much greater than in plasma, as blood passes through capillaries,
HCO3- diffuses into the plasma. Since protein anions cannot cross the plasma membrane and sodium
potassium pump also prevents diffusion of Na+ and K+, to maintain the electrical neutrality, chloride
ions diffuse into the RBCs to replace HCO 3- . This process is called "chloride shift." and it is the
principal cause of high RBC chloride content in venous blood.

Choice A: Spectrin is a structural component of the membrane of RBes that gives the cells
flexibility. Abnormalities of spectnn result in abnormal red blood cell shape and susceptibility to
hemoysis

Choice B: Na/K ATPase is an important ion pump responsible for the maintenance of ionic
concentration gradients across the plasma membrane. This energy-requiring pump is electrogenic in
nature because it extrudes three sodium ions for every two potassium ions that enter the cell. This
creates negative intracellular potential. Na/K ATPase is not responsible for movement of chloride
ions.

Choice D: 2,3-DPG is present in small quantities in most of the cells. However in erythrocytes, its
concentration is high. In RBCs, 2,3-DPG combines with hemoglobin and decreases its affinity for
oxygen, which facilitates oxygen delivery to the tissue. 2,3-DPG itself is not responsible for
chloride shift.

Choice E: Glucose-6-phosphate dehydrogenase is the first enzyme in the pentose phosphate


pathway. Patients with glucose-6-phosphate dehydrogenase deficiency present with episodic
hemolysis induced by oxidative stress.

Educational Objective:
Carbonic anhydrase activity within erythrocytes forms bicarbonate from CO 2 and water. Much of
the bicarbonate ions diffuse out of the RBC into the plasma. To maintain electrical neutrality of
erythrocytes, chloride ions diffuse into RBC to replace the lost bicarbonate ions. This process is
called 'chloride shift," and is the principal cause of high RBC chloride content in venous blood
Question 6

A group of investigators are studying the effect of various factors on the shape of the oxygen-
dissociation curve for hemoglobin.

The shift of the red curve towards the blue curve in the above graph would most likely be caused by
which of the following?

Answers
A. Hypothermia
B. Adaptation to high altitude
C. Hypoventilation
D. Severe hypoxemia
E. Polycythemia

Explanation

Ability of hemoglobin to carry oxygen from the alveolus to various organs and its subsequent
delivery to tissues is regulated by factors that determine the final shape of the “oxygen dissociation
curve”. The oxygen dissociation curve of hemoglobin describes the relationship between the partial
pressure of oxygen (x-axis) and the oxygen saturation of hemoglobin (y- axis). Hemoglobin binds to
oxygen more avidly as pO2 rises. An abrupt increase in oxygen-binding affinity of hemoglobin as
oxygen begins to bind to the hemoglobin molecule is responsible for the sigmoid shape of the
oxygen Hb dissociation curve. More and more oxygen molecules bind to hemoglobin as partial
pressure of oxygen increases. This occurs until the maximum amount of oxygen that can be bound
is reached. As this limit is reached, very little additional binding can take place.
The partial pressure of oxygen in blood at which hemoglobin is 50% saturated is known as the P 50.
The P5o is a conventional measure of hemoglobin affinity for oxygen. A shift of the oxygen
dissociation curve to the left occurs when hemoglobin has an increased affinity for oxygen; whereas
a shift to the right occurs when hemoglobin has a decreased affinity for oxygen. Hypothermia,
alkalosis and decreased levels of 2.3,diphosphoglycerate in RBC shift the dissociation curve to the
left.
Choice A: Hypothermia will shift the curve to left and a lower pO2 will be required to bind to a
given amount of oxygen.

Choices B, C and E: 2,3-diphosphoglycerate, or 2,3-DPG, is an organophosphate that is formed in


erythrocytes during glycolysis. Production of 2,3-DPG can increase in several conditions that
produce diminished peripheral tissue oxygen availability. Examples include hypoxemia, chronic
lung disease, anemia, and congestive heart failure. High levels of 2,3-DPG shift the curve to the
right because they decrease the affinity of hemoglobin for oxygen. In fact, high altitude adaptation
results from a decreased environmental pO2 causing an increase in 2,3-DPG levels. Hypoventilation
and chronic anemia will also shift the dissociation curve to right, thereby facilitating increased
oxygen delivery to tissues. Polycythemia is unlikely to cause shift in oxygen dissociation curve.

Educational Objective:
Left shift of the hemoglobin oxygen dissociation curve indicates increased hemoglobin-O 2 affinity
and can be caused by increased pH, decreased 2,3-DPG levels and decreased temperature. In this
situation a higher pO2 is required for hemoglobin to bind to a given amount of oxygen. A left-shift
of the oxygen-dissociation curve means that oxygen is relatively less available to tissues.
Question 7

A 10-year-old male has hemolytic anemia. One of his erythrocytic enzymes is found to be defective.
Normal version of this enzyme has 190 residues. Patient’s enzyme contains 156 residues. Cause of
this defect is identified as a point mutation in exon 2. Which of the following mRNA code is likely
to have changed in this case?

Answers

A. UCA  UGA
B. UUU  UUA
C. UAA  UAG
D. CUU  AUU
E. UAC  CAC

Explanation:
DNA molecule serves as a template for formation of mRNA. After the requisite splicing if rnRNA
that involves removal of non-encoding introns, the mRNA molecule is transported to the cytoplasm
where it is translated. Because 64 possible combinations of the four bases of DNA exist, there are
64 possible codons. Many amino acids have more than one possible codon. For instance, both UUU
and UUA code for the amino acid phenylalanine. Some codons code for the termination of synthesis
of the polypeptide chain and are called stop codons. Most of the stop codons consist of UAA, UAG,
and UGA bases.
Mutations can alter protein structure. If mutation is seen in a mRNA transcript that codes for an
enzyme, the function of that enzyme would be compromised. In the case described above, it is seen
that the implicated protein is significantly shortened. This indicates a premature placement of a stop
codon in the concerned mRNA molecule. Of the possible answers mentioned above, a single base
change (point mutation) in UCA codon would convert it to a stop codon UGA. The introduction of a
stop codon in middle of a protein sequence is called a nonsense mutation (Choice A).

Choice B: UUU and UUA both code for phenylalanine and this mutation will not after the protein
structure either, as it is also a silent mutation

Choices C: Changing UAA to UAG would not modify the structure of the protein because both of
these codons are stop codons. This type of mutation is called a silent mutation.

Choices D: Changing CUU, which codes for leucine to AUU (coding for isoleucine) will result in
an amino acid change at one position. The function of this protein may be altered depending on a
variety of factors but the ultimate size of the protein will remain the same. This type of mutation is
celled a missense mutation.

Educational Objective:
UGA, UAG, and UAA are stop codons. Introduction of stop codons at an inappropriate site will
produce premature termination of mRNA translation. This result in truncation of the polypeptide
chain and such mutations are called nonsense mutations.
Question 8

A 30 - year-old male is brought to ER in a stuporose condition. His history reveals attempts at


suicide, twice in the past. This evening a neighbor found him in a closed garage with the car
running. On examination the patient is semi-conscious. He has no known drug allergies. The toxic
substance responsible for the patient’s present condition is likely to affect hemoglobin by:

Answers
A. Oxidation of the iron moiety
B. Alteration of porphyrin ring of the hemoglobin molecule
C. Denaturing globin molecules of hemoglobin
D. Iron within heme binds preferentially to carbon monoxide
E. Diminishing partial pressure of oxygen

Explanation:
The patient is likely to be suffering from carbon monoxide poisoning. Carbon monoxide (CO) is
generated as a byproduct of incomplete combustion of hydrocarbons. Running vehicle present in
poorly ventilated spaces may produce CO poisoning. Compared to oxygen, CO has 220 times
greater affinity for hemoglobin. Inhaled CO rapidly diffuses across the alveolar membranes and
binds to iron contained in hemoglobin forming carboxyhemoglobin. This decreases the oxygen
saturation of blood. Carbon monoxide also inhibits release of oxygen from hemoglobin in tissues by
altering the conformation of hemoglobin into the relaxed form, which has high affinity for oxygen.
All these factors result in a left shift of the oxygen dissociation curve and tissue hypoxia. Treatment
of carbon monoxide poisoning consists of administration of 100% hyperbaric oxygen.

Choice A: Iron bound to heme is in reduced form {Fe++). Methemoglobin is formed when iron is
oxidized to the ferric form (Fe+++). Methemoglobin is incapable of binding to oxygen.

Choices B and C: Heme is an iron containing protoporphyrin. Ferrous iron is held in center of the
heme portion of the hemoglobin molecule by four nitrogen atoms of the pophyrin ring. Although
CO binds to iron, it does not alter the porphyrin ring.

Choice E: The partial pressure of oxygen is dependent on the concentration of oxygen dissolved in
plasma and not on actual concentration or the content of oxygen in hemoglobin. CO poisoning does
not alter the amount of oxygen dissolved in plasma.

Educational Objective:
CO binds to hemoglobin with an affinity that is 220 times greater than oxygen. CO shows
competitive binding with hemoglobin. Due to greater affinity of hemoglobin to CO as compared to
oxygen, presence of CO with prevent binding of oxygen to hemoglobin
Question 9
A 6-month-old African American baby is brought to the hospital for annual check up. He is
underweight (40th percentile for weight) and underdeveloped (54th percentile for height). His
mother is healthy, consumes no alcohol, nor is on any medication. But the baby is exclusively
breast-fed. Which of the following vitamin supplements need to be given to the baby for his present
condition?

Answers

A. Vitamin D
B. Vitamin A
C. Folic acid
D. Thiamine
E. Riboflavin
Explanation:
Breast milk is a complete diet for a developing baby, as it contains all the required proteins,
carbohydrates, fats, vitamins and trace minerals along with immunoglobulin, amino acids and
enzymes. Except vitamins D and K, all vitamins and trace elements are present in breast milk in
required amounts. Although, vitamin D content of breast milk is low, adequate levels of Vitamin D
can be synthesized in body, if the baby is given regular sun exposures. Because melanin in skin acts
as a barrier to UV rays, darker skinned infants require greater sunlight exposure to produce adequate
amounts of vitamin D. UV rays in sunlight convert 7-dehydrocholesterol in skin to vitamin D. For
this reason, exclusively breastfed dark-skinned infants who are not exposed to ample sunlight
should receive vitamin D supplementation.

Choice B: Although the content of vitamin A in liver is low at birth, it increases rapidly because of
presence of large amounts of vitamin A in colostrum and breast milk.

Choices C, D and E: Human breast milk normally provides adequate amounts of folic acid.
thiamine and riboflavin.

Educational Objective:
Content of vitamins D and K in breast milk is insufficient to meet the nutritional demands of a
developing newborn. Vitamin D deficiency may develop in exclusively breast fed infants if
adequate sun exposure is lacking. Dark-skinned infants are especially at risk because they require
greater amount of sun exposure to generate adequate amounts of vitamin D. Deficiency of vitamin
K is taken care of at time of birth to prevent hemorrhagic disease of the new born. This is done by
routine parenteral administration of vitamin K to new born.
Question 10

An 8-year-old African American male is brought to your office for a routine check-up. His mother
remarks that he often seems uninterested in playing with his peers and appears to "run out of breath
quickly." His medical records reveal that he has missed several pediatric vaccinations. He has been
hospitalized twice, once with a chest infection and at another instance with abdominal pain. The
patient mentions to you that occasionally his 'bones hurt.' Which of the following protein changes is
most likely to account for this patient's condition?

Answers
A. Phenylalanine deletion
B. Valine substitution for glutamic acid
C. Phenylalanine substitution for proline
D. Valine substitution for lysine
E. Early termination of polypeptide synthesis

Explanation:

This patient is exhibiting signs and symptoms of sickle cell anemia. Sickle cell anemia is a
hemoglobinopathy that typically affects patients of African ancestry. Point mutation in the 6 th codon
of beta-globin gene results in substitution of valine {hydrophobic) for glutamic acid (hydrophilic)
and produces this condition. Incorporation of this abnormal beta globin protein into hemoglobin
results in formation of hemoglobin S (HbS). HbS polymerizes in conditions of low oxygen tension,
which causes sickling and hemolysis of erythrocytes with resultant vascular occlusion. This patient's
poor exercise tolerance and exertional dyspnea are due to anemia. His history of acute chest
infection, abdominal and bone pain are due to vaso-occlusive events in the lungs, spleen and bone
respectively

Choice A: Phenylalanine deletion is the most common cause of cystic fibrosis. It occurs due to a 3
bp deletion in a gene located on chromosome 7. This results in absence of phenylalanine at amino
acid position 508 of the cystic fibrosis gene protein product. It is the commonest fatal genetic
disease amongst Caucasians

Choice E: Early termination of polypeptide chain synthesis (nonsense mutation) will produce a
truncated protein

Educational Objective:
Exertional dyspnea, pneumonia resulting in life-threatening acute chest syndrome, and recurrent
abdominal and bone pain are clinical features of sickle cell anemia. Sickle cell anemia results from
a point mutation that causes valine substitution for glutamic acid in the 6 th codon of the beta globin
chain of hemoglobin.
Question 11

A 42-year-old female presents with complaints of difficulty in swallowing, severe fatigue and
progressive exertional dyspnea. Physical examination shows skin pallor, pale conjunctiva and nail
abnormalities. A photograph of her nails is shown below. She should be treated with which of the
following medications?

Answers

A. Vitamin B12
B. Pyridoxine
C. Vitamin E
D. Folic acid
E. Iron preparations
F. Erythropoeitin

Explanation:
Some clinical features are seen in all forms of anemia. They include malaise, increased fatigability,
skin pallor, decreased exercise tolerance and congestive heart failure. Certain other features like
dysphagia and koilonychia (spoon shaped nails) are more specific for iron deficiency anemia. The
above illustration depicts spoon shaped nails. Dysphagia in a patient of iron deficiency anemia is
due to formation of esophageal webs. It is called as Plummer-Vinson or Patterson Kelly syndrome.
The characteristic peripheral blood picture in iron deficiency anemia shows presence of microcytic
hypochromic RBCs. Iron deficiency anemia is usually treated with an oral iron preparation. As iron
absorption is enhanced by presence of vitamin C, it is sometimes added in the treatment schedule.
Choices A and D: Deficiency vitamin B12 and/or folic acid produces megaloblastic anemia

Choice B: Pyridoxine is required as a cofactor for the first step in heme synthesis. Therefore
pyridoxine deficiency causes also decreases heme synthesis. Such patients show microcytic,
hypochromic pyridoxine-responsive anemia - sideroblasfic anemia.

Choice F: In chronic renal diseases, formation of erythropoietin is diminished. Hence erythropoietin


preparations are commonly used to treat anemia associated with chronic renal failure.

Educational Objective:
The symptoms of difficulty in swallowing (dysphagia) and disfigured fingernails (spoon nails or
koilonychia) are specific for iron deficiency anemia.
Question 12

An 18-year-old male presents to the physicians office with complaints of heaviness in the
abdominal region. His spleen was found to be enlarged. His RBCs were tested for several enzymes,
in which the assays revealed a low pyruvate kinase activity. In this patient, which of the following is
the most likely cause of spleenomegaly?

Answers

A. Intracellular accumulation of certain abnormal substances.


B. Passive venous congestion.
C. Infestation with malarial parasite.
D. Work hypertrophy.
E. Neoplastic disorder of spleen.
F. Idiopathic spleenomegaly
Explanation:
Along with lymph nodes and thymus, spleen is a major organ of the reticuloendothelial system. It
contains approximately 25% of body’s lymphoid tissue. It also plays a scavenging role in the body
and is concerned in removal of senescent and defective red blood cells. Other functions of spleen
include antibody production and B cell affinity maturation. It also removes antibody-coated bacteria
and other opsonized matter from the circulation. An increase in any of these normal functions may
result in spleenomegaly. A mature RBC does not have a nucleus, ribosomes and mitochondria and
thus its ability to generate ATP is limited. In RBCs, ATP is generated by the Embden-Meyerhof
pathway (glycolysis), and whatever small amounts of energy are liberated by this is utilized in
maintaining the humble metabolic demands of this cell. Pyruvate kinase is an enzyme in Embden-
Meyerhof pathway. It converts phosphoenolpyruvate to pyruvate resulting in generation of a
molecule of ATP. Any deficiency of glycolysis in red blood cells leads to hemolysis because of
insufficient production of ATP and defective maintenance of erythrocytic cell membrane. Such cells
develop rigid cell membranes and are destroyed during their passage in spleen. Excessive
erythrocyte destruction by the spleen causes splenomegaly due to work hypertrophy (choice D).

Choices A, B, C and E: The other choices listed above also result in splenomegaly. However in
these conditions, spleenomegaly is not due to work hypertrophy. In Niemann.Pick disease,
accumulation of sphingomyelin and glucocerebrosides is responsible for spleenomegaly. In passive
venous congestion, dilatation and congestion of the spleenic sinusoids is responsible. As mentioned
earlier, spleen also participates in fighting microorganisms. Thus spleenomegaly is a frequent
accompaniment of many acute and chronic infectious disorders. In these diseases hyperplasia of
lymphoid cells produces spleenic enlargement. Disorders such as leukemia and lymphoma also
result in spleenomegaly because of neoplastic proliferation of lymphoid cells or due to
extramedullary erythropoeisis that occurs in these diseases.

Educational Objective:
Pyruvate kinase deficiency causes hemolytic anemia due to failure of glycolysis and resultant
failure to generate sufficient ATP. Due to resultant energy deficiency, RBC is not able to maintain
the normal plasticity of its cell membrane, which becomes rigid and is damaged during its normal
passage within the spleen.
Question 13

A 5-days-old infant born to a 22-year.old female demonstrates a good appetite despite some weight
loss. Physical examination of this infant is within normal limits. Which of the following hemoglobin
pattern is likely to be the predominant form in this infant?

A. Α2ß2
B. Α2δ2
C. Α2γ2
D. ζ2ε2
E. γ4

Explanation:
Predominant hemoglobin (Hb) in last seven months of gestation is fetal Hb (HbF). It consists of two
alpha and two gamma globin chains. HbF comprises 70-95% of all hemoglobin in the newborn and
is gradually replaced by hemoglobin A (adult hemoglobin) during the first six months of post-natal
life.
Choice A: it depicts normal hemoglobin found in adults Adult hemoglobin (Hb A) has two alpha and
two beta globin subunits.
Choice B: It describes the structure of hemoglobin A2. It is a normal variant of human hemoglobin
that consists of two alpha and two delta chains. In an adult it comprises 2 to 5% of hemoglobin.
Choice D: It is called hemoglobin Gower. It is the earliest hemoglobin found in fetus and consists
of 2 zeta and 2 epsilon globin chains. It is gradually replaced by HbF by 10 to 12 weeks of
gestation.
Choice E: Hemoglobin Bart consists of four gamma globin chains. It is found in homozygous
alpha- thalassemia, where a genetic disorder prevents synthesis of alpha globin chains.
Homozygous alpha thalassemia is incompatible with life and is usually associated with hydrops
fetalis.

Educational Objective:
The predominant form of hemoglobin in fetal life is HbF, which consists of 2 alpha and 2 gamma
globin chains. It has a high affinity for oxygen. It is gradually replaced by HbA during early post
natal life. However in certain disease conditions like beta thalassemia, its synthesis never stops
completely and helps the body to prevent development of extreme anemia.
Question 14

A 30-year-old African American female has a 3-year-old child with sickle cell anemia. She is
concerned that her next child may be affected by this disease as well. She herself does not have any
symptoms. She has remarried two years ago and wants to have a prenatal checkup regarding this
disease. Which of the following is the best initial test that should be offered to this woman?

Answers
A. Maternal hemoglobin electrophoresis
B. Karyotype analyses of both parents
C. Western blot analysis of both parents
D. Chorionic villous sampling during pregnancy
E. Hemoglobin electrophoresis of father’s blood sample.

Sickle cell anemia is an autosomal recessive disorder chararacterized by presence of abnormal


hemoglobin (HbS) in RBCs of the affected patients. In HbS, the glutamic acid in the 6 th position on
the beta chain is replaced by valine. In order to develop the disease; the affected person must inherit
abnormal genes for hemoglobin S (HbS) from both of his parents. For this to happen, each of their
parents must carry the gene for HbS. A person carrying HbS gene on one of his chromosomes is
said to have a sickle cell trait. He does not have sickle cell disease. When both parents have sickle
cell trait, any offspring resulting from such a pairing will have a one in four chance of developing
sickle cell disease (see diagram above). Because this woman already has one child with sickle cell
anemia, she obviously is a carrier. Hence hemoglobin electrophoresis of her new husband should be
performed to see whether he too is a carrier of HbS gene (Choice E).

Choice A: It is not necessary to perform hemoglobin electrophoresis in the mother because presence
of sickle cell disease in one child indicates that she has sickle cell trait.

Choice B: A karyotype is a visual analysis of all the chromosomes in a cell in which large
chromosome segments are visualized by chromosomal banding. Sickle cell disease results due to a
single nucleotide mutation and not from any abnormality in overall chromosome structure.
Karyotype analysis is done more often to detect abnormalities in number of chromosomes or for
detection of gross abnormalities in a particular chromosome.

Choice C: Western blot analysis utilizes electrophoresis to separate different proteins by their size
and electrical charge. Individual proteins are subsequently analyzed using specific antibodies.
Hemoglobin electrophoresis differs from Western blot analysis because it only involves analysis
type of hemoglobin by electrophoresis.

Choice D: Chorionic villi are part of the placenta and thus their sampling can be done only after
conception has occurred and the fetus has reached a certain size. It cannot be done before pregnancy
is established. Hence it is not an investigation which is helpful for a parent who wishes to determine
his or her risk of carrying the sickle cell gene.

Educational Objective
In order for a child to have sickle cell disease, both parents must be carriers for the sickle cell gene.
The carrier status of a prospective parent can be established by hemoglobin electrophoresis. This
usually shows presence of about 60% HbA, 40% HbS, normal HbF and normal to slightly increased
HbA2, up to 4.5%. Some recent techniques utilizing molecular biologic tests are also described.
They include detection of sickle cell point mutation by either Southern blot or restriction enzyme
digestion of beta globin PCR products. Dot blot techniques have also been found useful.
Question 15

A 30 year old male is detected to have anemia on routine check up. He was advised iron
supplements for three weeks. On follow up it is seen that no improvement in hemoglobin levels has
taken place. His investigations during follow up are as follows:

 Hemoglobin: 8.5 gms/dL


 RBC count: 3 .9 million/cmm
 MCV: 64 fl
 Total WBC count: 6500 cell/cmm
 Platelet count: 180,000/cmm
 HbF: 5%

What defect is likely to be present in this patient?

Answers

A. Unstable mRNA activity


B. Abnormal protein folding
C. Abnormal protein solubility
D. Defective metabolic enzyme
E. Cell membrane instability
F. Defective DNA synthesis
.
Explanation:
This patient has microcytic anemia with increased concentration of HbF (Normal levels: 0.1% -
0.5%). His peripheral blood smear shows hypochromic erythrocytes along with target cells. These
findings in an asymptomatic adult patient of Mediterranean origin suggest a diagnosis of beta-
thalassemia minor. Iron supplements will not benefit these patients.
Beta-thalassemia is the most common thalassemia in persons of Mediterranean descent. Alpha-
thalassemia is more common in Southeast Asian individuals. Normal adult hemoglobin (HbA)
contains two alpha chains and two beta chains (α2ß2). In beta- thalassemia, there is decreased
production of ß–globin chains of hemoglobin molecule with normal production of α-globin chains.
197 different mutations have been identified as a cause of ß thalassemia. Most are associated with
single base substitution that produce defect in promoter activity. RNA processing, splicing or
translation of mRNA of ß-globin gene is abnormal in these patients. In patients of ß thalassemia,
diminished production of beta chains along with normal production of alpha chains and heme
moiety lead to increased formation of HbF and HbA2

Choice B: An example of protein folding defect producing disease is alpha-1 antitrypsin deficiency.

Choice C: Protein solubility is abnormal in sickle cell disease. Abnormal HbS present in these
patients shows decreased solubility and it polymerizes in presence of low oxygen to produce
insoluble crystals.

Choice D: An example of impaired intracellular enzyme activity is glucose-6-phosphate


dehydrogenase deficiency as a cause of hemolytic anemia.

Choice E: Red cell membranes are defective nad hence unstable in diseases like Hereditary
spherocytosis, hereditary elliptocytosis and paroxysmal nocturnal hemoglobinuria.

Choice F: Abnormal DNA synthesis as a cause of anemia is seen with Vitamin B 12 and folic acid
deficiency.

Educational Objective:
ß-thalassemia results from mutations that cause defective mRNA processing. This leads to
deficiency of ß-globin chains required for hemoglobin synthesis. ß -thalassemia minor is typically
an asymptomatic disorder Laboratory tests will show mild microcytic hypochromic anemia with
presence of target cells.
Question 16

The oxygen hemoglobin dissociation curve that relates percentage saturation of the oxygen carrying
power of hemoglobin to pO2 is sigmoid shaped, as illustrated below. During the transition from
point 1 to point 2 on this curve, the hemoglobin molecule is most likely to release which of the
following?

Answers
A. Heme
B. Chloride
C. Phosphate
D. Protons
E. Oxygen

Explanation:

At a very low p02, the hemoglobin molecule is fully deoxygenated. Oxygen-binding affinity
increases as the p02 increases. Combination of the first heme in the Hb molecule with oxygen
increases the affinity of the second heme for O 2, and oxygenation of the second increase the affinity
of the third heme and so on. Hb molecule binds to more 0 2 molecules as the partial pressure of
oxygen increases. When hemoglobin becomes saturated with oxygen, very little additional binding
occurs and the curve levels out. This is illustrated in the graph above and the same phenomenon
occurs in lungs.
The decrease in O2 affinity of Hb when the pH of the blood falls is called Bohr effect. It is caused
by ionizable histidine side chains at the N-terminal alpha amino group of the alpha and beta
hemoglobin subunits that bind protons [H+] with high affinity when hemoglobin is deoxygenated.
The increased concentration of protons at the tissue level causes the amino groups to be protonated
(charged) and able to form ionic bridges (see diagram). Formation of these salt bridges stabilizes the
deoxygenated form of hemoglobin and decreases hemoglobin's affinity for oxygen thereby
facilitating the release of oxygen at the tissue level. In other words deoxyhemoglobin binds protons
more avidly than oxyhemoglobin. The opposite effect occurs at the level of the lung; the increase in
p02 at the alveolar level increases the binding of oxygen to hemoglobin and causes release of
protons. The concentration of protons is greater at the tissue levels because [H+] are formed during
metabolic activity of the tissues. (Choice D)

Choice A: Heme is not released from hemoglobin in the context of oxygen loading or dissociation:
however it is released from hemoglobin during normal destruction of aged red blood cells by the
spleen.

Choices B and C: Neither chloride, nor phosphate are transported in any important manner by
hemoglobin. Chloride is shifted out of erythrocytes in venous blood by bicarbonate, and it re-enters
RBCs in the arterial blood.

Choice E: Oxygen is released from hemoglobin in the tissues most commonly where conditions
favor proton binding and the release of 0 2. In the tissues the pH is lower, temperature is greater, and
the concentration of 2.3 DPG is also higher than in the lung All of these conditions favor a
decreased affinity of hemoglobin for 02

Educational Objective:
In the lungs, hemoglobin binds to oxygen and releases [H+]; while in the tissues it releases 0 2 and
acquires [H+]; Deoxyhemoglobin is stabilized by bonding of 2.3-DPG to the beta chains of
deoxygenated Hb and by salt bridges between N-terminal histidine residues in each globulin.
Question 17

A 30-year-old Caucasian female with recurrent abdominal pain has had marked improvement in her
symptoms after intravenous administration of a heme preparation. Rapid improvement of symptoms
in this patient is most likely due to significant repression of.

Answers
A. ALA synthase
B. ALA dehydratase
C. Ferrochelatase
D. Bilirubin glucuronyl transferase

Explanation

Heme (as contained in hemoglobin molecule) is an iron containing porphyrin. Most body cells can
synthesize heme but bone marrow and liver are the most active. The pathway of heme synthesis is
shown below.

Porphyrias comprise a group of inherited and acquired disorders of heme biosynthesis due to
deficiency of enzymes in heme biosynthesis. Due to this, there is excess production of precursors
formed in the steps prior to the enzyme defect. Porphyrias can be broadly classified as either hepatic
or erythropoietic, depending on the site of the enzymatic deficiency.

Heme in liver is used in cytochrome p450 enzyme system, while heme in bone marrow is generated
for hemoglobin use. The synthetic pathway in liver and bone marrow are regulated differently
because the heme generated by these two separate tissues serves different functions.
Clinical manifestations of porphyria result from accumulation of precursors of porphyrins in blood,
tissues, and urine. Acute attacks of intermittent hepatic porphyria can be precipitated by
administration of drugs such as phenobarbital. gresiofulvin and phenytoin. Additionally, alcohol and
a low caloric diet can induce an acute attack of porphyria. All of these factors precipitate porphyria
symptoms by decreasing the hepatic concentration of heme, which causes an increase in hepatic
ALA synthase activity and leads to increased formation of 5-aminolevulinic acid and
porphobilinogen. This happens because, normally the concentration of heme, has a negative
feedback effect on synthesis of ALA synthase. Increased ALA synthase activity leads to increased 5-
aminolevulinic acid and porphobilinogen levels. In patients with acute intermittent porphyria, 5-
aminolevulinic acid (ALA) and porphobilinogen accumulates and high concentrations of these
intermediates are responsible for acute abdominal pain and neurological symptoms. Diagnosis of
acute intermittent porphyria is made by demonstrating elevated levels of ALA and porphobilinogen
during an acute attack.

Choices B and C: ALA dehydratase and ferrochelatase are inhibited by lead and not by heme.

Choice D: Bilirubin glucuronyl transferase is a hepatic enzyme that is responsible for the
conjugation of bilirubin (a byproduct of heme catabolism) with glucuronic acid. A decrease in
glucuronyl transferase activity results in unconjugated hyperbilirubinemia.

Educational Objective:
In acute intermittent porphyria, heme (in various forms such as hematin or heme arginate) is used
for relieving pain as it inhibits activity of ALA synthase and thereby prevents accumulation of
intermediate metabolites like 5 -aminolevulinic acid and porphobilinogen.
Question 18

A 58-year-old male is brought to ER with complaints of anxiety, weakness, dyspnea and headaches.
Physical examination reveals cyanosis that is not corrected by oxygen administration. Subsequently
it was discovered that he has been accidentally exposed to large quantities of nitrites. Of the
following parameters, which of the following is most likely to be normal in this patient?

Answers
A. Partial pressure of oxygen in arterial blood
B. Oxygen content of the arterial blood
C. Oxygen carrying capacity of the arterial blood.
D. Oxygen delivery to the peripheral tissues
E. Bound fraction of oxygen in the arterial blood

Explanation:

Iron in hemoglobin is normally in present in a reduced ferrous [Fe++] state. Oxidation of the ferrous
iron to ferric [Fe++] state leads to the formation of methemoglobin. Methemoglobin is not able to
bind oxygen. And levels of 50 to 60% are generally fatal. However affinity of any residual ferrous
iron in the hemoglobin tetramer is increased, thereby causing a leftward shift of the oxygen-
dissociation curve. Nitrites cause poisoning by oxidizing the heme iron to the ferric state. Similarly
other oxidizing compounds may also produce methemoglobinemia.
In patients with methemoglobinemia, the partial pressure of oxygen in blood (Choice A) is normal
because the amount of oxygen dissolved in the plasma is not altered. Hence diagnosis of
methemoglobinemia should be suspected in any patient of cyanosis whose pO 2 is sufficiently high
to produce full saturation of Hb. Because methemoglobin is unable to carry oxygen, a state of
functional anemia is induced. Thus levels greater than 15% induce a state of cerebral ischemia.
Choices B, C, D and E: Methemoglobin cannot bind to oxygen and therefore the oxygen content
and oxygen carrying capacity of the arterial blood will decrease. The bound fraction of oxygen and
oxygen delivery to the peripheral tissues will also decrease because of the inability of
methemoglobin to carry oxygen

Educational Objective:
Methemoglobin causes dusky discoloration to the skin (similar to cyanosis), and because
methemoglobin is unable to carry oxygen, a state of functional anemia is induced. The blood partial
pressure of 02, however, will be unchanged in this condition because oxygen's partial pressure is a
measure of 02 dissolved in the plasma and is not related to hemoglobin function. Presence of
methemoglobin should be sought by spectroscopy when cyanosis exists despite absence of
circulatory or respiratory disorders. When blood of a patient suffering from methemoglobinemia is
collected in a test tube, it remains dusky brown in color despite adequate exposure to air.
Methemoglobinemia is usually produced due to exposure to oxidizing agents like nitrites,
naphthalene, primaquine, sulphonamides, aniline etc.
Question 19

If the mechanism of energy-dependent transport of organic anions across hepatocellular membranes


is inhibited, which of the following effects is most likely to occur?

Answers
A. Urobilinogen absorption in gut will increase
B. Urine will have increased concentration of bilirubin
C. Unconjugated bilirubin levels would rise in blood
D. Haptoglobin concentration in serum will decrease.
E. Stools would be more yellow in color.

Unconjugated bilirubin formed from degradation of heme in reticuloendothelial cells enters


hepatocytes by passive diffusion and by receptor mediated endocytosis. Conjugated bilirubin is
secreted from hepatocytes into the bile canaliculi by an active energy dependant process. Inhibition
of active secretion of conjugated bilirubin into the bile canaliculi would increase blood levels of
conjugated bilirubin because of regurgitation. There are two important physiologic differences
between conjugated and unconjugated bilirubin. Unconjugated bilirubin is virtually insoluble in
water at physiologic pH and is tightly complexed to serum albumin within the circulation. This form
cannot be excreted in urine even when blood levels are high. In contrast, conjugated bilirubin is
water soluble, nontoxic, and only loosely bound to albumin. Because of its water solubility excess
conjugated bilirubin in plasma can be excreted in urine.
Jaundice occurs when the equilibrium between bilirubin production and its excretion is disturbed. If
the active energy dependant process of bilirubin secretion is compromised, blood levels of
conjugated bilirubin will rise and more of it would be excreted in urine (Choice B).
Choice A: Bilirubin contained in bile reaches the gut, where it is broken down into urobilinogen by
the activity of gut flora. Increased urobilinogen absorption in the gut takes place when increased
amounts of bilirubin reach the gut to be broken down to urobilinogen. This is subsequently
reabsorbed and re-secreted by the hepatocytes. If energy dependant hepatocytic secretion is
impaired, as in the case above, lesser amounts of bilirubin will reach the gut and lesser amounts of
urobilininogen will form and reabsorbed.

Choice C: Levels of unconjugated bilirubin would rise in blood only if it is unable to enter
hepatocytes. As this process occurs through passive diffusion and by receptor mediated endocytosis,
the process will not be hampered if active transport mechanisms are compromised.

Choice D: Haptoglobin is an acute phase reaction protein whose concentration rises normally in
response to inflammation and tissue necrosis. It also has the property of binding to hemoglobin. Fall
of haptoglobin level is usually seen after intravascular hemolysis because haptoglobin-hemoglobn
complexes are cleared by cells of the reticuloendothelial system. Hence decreased levels of serum
haptoglobin are seen in hemolytic anemias.

Choice E: Normally urobilinogen formed in gut - by the action of intestinal microbial flora - is
converted to stercobilin, which imparts yellow color to the stools. If less or no bilirubin is reaching
the gut as in obstructive jaundice or due to defect in bilirubin secretion from hepatocytes, stools
would be lighter in color.

Educational Objective:
The liver takes up unconjugated bilirubin from circulating blood through a passive process. It
secretes conjugated bilirubin into bile canaliculi and further into biliary tract by an energy
dependant active process. Unconjugated bilirubin is virtually insoluble in water at physiologic pH
and is tightly complexed to serum albumin in the circulation. This form cannot be excreted in the
urine even when its blood levels are high.
Question 20
A 60-year-old impoverished man is having irregular, infrequent and unbalanced meals for last three
years. He is also an alcoholic. He presents to a clinic with symptoms of certain vitamin deficiencies.
What vitamin deficiencies are not likely to be present in this individual?

Answers
A. Vitamin A
B. Vitamin D
C. Folic acid
D. Vitamin K
E Thiamine
F Riboflavin
G Cobalamin

Explanation:
Deficiency of vitamins develops either due to inadequate intake, insufficient synthesis within body
or due to presence of substances that antagonize the action of these vitamins (e.g. anti-metabolite
drugs). Manifestations of vitamin deficiency appear, when body stores are totally depleted. Hence in
the above case, deficiency of those vitamins which are either synthesized within body or whose
stores last for more than three years will not be seen will not be seen. More than 90% of the body’s
vitamin A reserves are in the liver, mainly in the peri-sinusoidal cells of Ito. These stores are
sufficient to last about 6 months. Vitamin D is stored in adipose tissue, but vitamin D deficiency can
develop over the course of months in the face of inadequate dietary intake and minimal sunlight
exposure The small quantity of vitamin K normally stored in the liver is only sufficient to meet the
body's biochemical requirement's for 1-3 weeks However, severe vitamin K deficiency usually does
not develop from dietary deprivation alone, since bacteria in the largo intestine are able to produce
functional forms of vitamin K. With the exception of vitamin B12 {cobalamin), all water soluble
vitamins (including folic acid, thiamin, and riboflavin) are flushed from the body relatively quickly.
Cobalamin however, is stored in significant quantities in normal liver and its stores last for several
years after complete dietary deprivation.

Educational Objectives:
With the exception of vitamin B12, the body's stores of most water soluble vitamins are rapidly
depleted without adequate dietary intake. Liver stores of vitamin A last about six months without
dietary intake. Fat stores of vitamin D can last a few months in absence of dietary intake or sunlight
exposure. Severe vitamin K deficiency rarely results from poor dietary intake because colonic
bacteria produce functional forms of vitamin K.
Question 21
A 16-year-old African American male complaint of passing dark colored urine on being prescribed a
sulphonamide for upper respiratory tract infection. His RBCs were found to have low activity of
glucose-6-phosphate dehydrogenase (G6PD) enzyme. Deficiency of which other enzyme, amongst
the following, is likely to produce similar effects?

Answers

A. Glutathione reductase
B. Pyruvate kinase
C. Biphoshosphoglycerate mutase
D. Enolase
E. Hexokinase

Explanation:

Red blood cells do not have mitochondria or a nucleus: therefore metabolism of glucose in these
cells occurs via glycolysis and hexose monophosphate (HMP) shunt. HMP shunt provides the
reducing agent NADPH to prevent oxidant damage to RBCs and is the sole means of providing this
molecule to RBCS.. In the initial oxidative portion of the HMP shunt, glucose 6-phosphate is
converted to 6-phosphogluconolactone and one molecule of NADPH is generated. This reaction is
catalyzed by glucose 5-phosphate dehydrogenase, a rate limiting step of HMP shunt. In an
accompanying reaction of the oxidative portion of the HMP shunt, 6-phosphogluconolectone is
hydrolyzed to ribulose 5-phosphate by the enzyme 6-phosphogluconate dehydrogenase producing
another molecule of NADPH.

As it is the only mechanism by which RBCs generate NADPH, absence of G6PD enzyme makes
erythrocytes extremely susceptible to destruction in face of an oxidant challenge.
In erythrocytes, hydrogen peroxide produced during normal metabolism is detoxified by glutathione
peroxidase. Glutathione is oxidized during this reaction. The regeneration of reduced glutathione is
carried out by the enzyme glutathione reductase using NADPH as an electron donor. NADPH in red
blood cells is produced solely by the HMP shunt, and this is how the HMP shunt contributes in
protecting red blood cells from oxidative stress. Defective generation of NADPH due to defects in
the oxidative portion of the HMP shunt will increase susceptibility of RBCs to oxidative damage;
similarly in glutathione reductase deficiency, hydrogen peroxide will not be detoxified will produce
a similar clinical picture (choice A).
Choices B, C and D: These are the enzymes of the glycolytic pathway which also are essential for
RBC survival. ATP generation by glycolytic pathway is essential in RBCs, because due to absence
of mitochondria, erythrocytes cannot oxidize glucose beyond the pyruvate stage. In situations of
decreased availability of ATP, erythrocytic membrane becomes stiff and deformed. These deformed
RBCs are prematurely cleared from circulation by the spleen or because of intravascular hemolysis.
The mechanism of hemolysis due to defective RBC glycolysis is different from hemolysis resulting
from defects in the HMP shunt in that hemolysis due to glycolytic defects causes a chronic
hemolytic anemia, whereas hemolysis due to HMP shunt defects takes place when RBCs are
subjected to an oxidant load. Pyruvate kinase deficiency constitutes more than 95% of cases of
hemolytic anemia due to defective RBC glycolysis.

Choice E: Transketolase catalyze the non-oxidative reactions of the HMP shunt and are primarily
designed to generate ribose 5-phosphate from fructose-phosphate and glyceraldehyde 3-phosphate
or the reverse. NADPH is not generated in the non-oxidative portion of the HMP shunt.

Educational Objective:
Glucose 6-phosphate dehydrogenase is an enzyme of HMP shunt. Its absence impairs glutathione
reduction due to failure of erythrocytes to produce NADPH, a step that is essential for
detoxification of hydrogen peroxide.. Glutathione reductase deficiency causes a similar clinical
picture and hence it effects are similar to G6PD deficiency.
Question 22

HbC disease is caused by point mutation in ß-globin chain of the hemoglobin molecule leading to
substitution of lysine for glutamic acid at position 6. Patients homozygous for HbC gene have mild
chronic hemolytic anemia. HbS disease (which also is caused by a mutation in ß globin gene at
position 6 with valine replacing glutamic acid) generally causes a more severe condition compared
to HbC disease because HbS disease

Answers
A. Impairs oxygen binding to the heme moiety
B. Impairs production of ß globin chains
C. Alters spectrin filaments of erythrocytes
D. Stabilizes iron moiety in ferric (Fe 3+) state
E. Allows hydrophobic interaction among hemoglobin molecules

Explanation:

In HbS a charged glutamic acid residue is replaced by a nonpolar hydrophobic valine residue at the
surface of the Hb molecule, exposed to water. This substitution replaces the polar GLU residue with
a non-polar one and generates a ‘sticky patch’ on the surface of the ß chain. When HbS is
deoxygenated, the sticky patch can bind to its complementary patch on another molecule of
deoxygenated HbS. This alteration causes aggregation of hemoglobin molecules under anoxic
conditions. After polymerization, HbS, initially forms a gel that changes into a meshwork of fibrous
polymers. This produces red blood cell distortion and inflexibility of erythrocytes. Blood viscosity
increases due to dehydration caused by K+ leakage and influx of Ca++ Such cells are known as
sickled RBCs. Sickling is promoted by conditions associated with low oxygen tension, increased
acidity, or low blood volume. All these factors promote occlusion of microvasculature and
production of microinfarcts.

In Hemoglobin C disease (which also is characterized by single amino acid substitution at the
position 6 of the beta globin chain) normally charged glutamic acid residue at position 6 is replaced
by a basic amino acid lysine. However as opposed to Hbs, crstal of HbC tend to melt at low pO2.
Persons with HbC generally have mild hemolytic anemia ,lack sickling or microinfarction as seen in
sickle cell disease.

Choice A: oxygen binding to heme moiety is seen in carbon monoxide poisoning.

Choice B: Impaired production of ß globin chains is seen in ß thalassemia.

Choice C: Altered spectrin filaments are seen in conditions like hereditary spherocytosis.

Choice D: Iron moiety is fixed in ferric state in conditions like poisoning with oxidizing agents or
toxicity of oxidizing drugs.

Educational objectives:
HbS disease occurs due to point mutation in ß globin chain of the hemoglobin molecule. Because of
mutation, red cells are less easily deformable due to polymerization of Hb molecules, especially in
conditions of low oxygen tension and low tissue pH. This tends to occlude the microvasculature of
the body with resultant microinfarction.
Question 23

Laboratory animals experiencing folic acid deficiency show marked increase in marrow erythroid
precursor cell production. However instead of maturing into adult erythrocytes, these precursors
undergo apoptosis (ineffective erythropoeisis). Provision of which of the following supplements
would enhance erythroid precursor cell population to reach normal maturity?

Answers

A. Thymidine
B. Homocysteine
C Cytosine
D. Glutamine
E. Cobalamin

Explanation:
Folate derivatives are essential for synthesis of purines, dTMP and methionine. As shown in the
following chemical reaction, thymididylate synthase catalyzes the methylation of dUMP to dTMP
(deoxythymidylate monophosphate). In this reaction, folate serves as an intermediate carrier of 1-
carbon fragments and during this process, fotate derivative, 5,10- methylenetetrahydrofolate is
converted to dihydrofolate.

5,10-methylenetetrahydrofolate + dUMP = dihydrofolate + dTMP.

This is the only de novo pathway for dTMP production. Thymidylate sythase is therefore essential
in regulating the supply of nucleotide precursors for DNA replication. Because of the role
mentioned above, DNA synthesis would be impaired if deficit of 5.10-methylenetetrahydrofolate
exists. One of the common consequences or reduced DNA synthesis is megaloblastosis. However a
salvage pathway using thymidine kinase exists (it normally accounts for 5-10% of dTMP synthesis),
which can be utilized for dTMP synthesis in face of folate deficiency. Activation of this pathway by
thymidine supplementation can partially compensate for diminished dTMP synthesis in condition of
folate deficiency (Choice A)

Choice B: Homocysteine levels are elevated in folate deficiency. Folate supplementation can reduce
the homocysteine levels.

Choice C: Although cytosine is a pyrimidine base, it is not a dTMP precursor. Therefore, cytosine
supplementation would not be expected to increase available dTMP levels.

Choice D: Glutamine is the major source of nitrogen in the synthesis of nucleotides. As such, it
contributes a nitrogen atom to the biosynthesis of dUMP. Later, 5,10-methylenetetrahydrofolate
donates one carbon group to dUMP, resulting in the formation of dTMP. In persons with severe
folate deficiency however any increases in dUMP stimulated by glutamine supplementanon would
typically not increase the available dTMP levels as effectively as thymidine supplementation would

Choice E: Although cobalamin supplementation can mitigate the impact of fotate deficiency on
erythropoiesis, it would not be as effective as directly increasing the available levels of dTMP.

Educational Objective:
Folate deficiency inhibits the formation of dTMP required for DNA synthesis. Hence folate
deficiency megaloblastosis and erythroid precursor cell apoptosis. Because thymidine
supplementation can moderately increase dTMP levels, it can reduce erythroid precursor cell
apoptosis.
Question 24
A solution containing a mixture of several proteins is subjected to electrophoresis and then
transferred to a solid support medium. Antibodies to a specific protein are then allowed to react with
these separated proteins. Next, a radioactive label that combines with this antibody-protein complex
is used to identify the specific protein under investigation. This type of hybridization analysis is
called

Answers

A. Northern blot
B. Southern blot
C. Western blot
D. South-Western blot
E. Microarray hybridization

Explanation

Western blotting is used to detect presence of specific polypeptide chain or protein from within a
mixture of sample proteins. First, denatured proteins are separated by gel electrophoresis.
Subsequently, separated proteins are transferred onto a nitrocellulose membrane. The target protein
is then identified using specific antibodies and radioactive probes For example, a serum sample
from a patient with suspected human immunodeficiency virus (HIV) infection can be analyzed by
Western blot to detect antibodies specific to the viral particles (choice C).

Choice A: Similar technique when used for analysis of RNA is called Northern blotting. First, a
sample containing a mixture of mRNA molecules is separated by gel electrophoresis. Separated
bands are transferred to a membrane and hybridized with a probe containing a nucleotide sequence
complementary to the mRNA of interest.

Choice B: Southern blotting in a method used to analyze DNA sequences. It was first discovered by
E.M. Southern. In this, DNA is first extracted from cells and is broken into smaller fragments using
restriction endonuclease enzymes. Fragmented DNA is subjected to electrophoresis and the DNA
fragments are transferred to a solid support medium - commonly nitrocellulose membrane. A
radiolabelled DNA probe containing a sequence complementary to the DNA fragment of interest is
hybridized with the separated bands. As mutations alter DNA fragment lengths, these can be
detected by finding an altered electrophoresis migration pattern.

Choice D: South-Western blotting is a technique that analyzes DNA-binding proteins using


principles of the Southern and Western blotting. In this, DNA is separated and blotted and further
incubated with protein solutions. This allows evaluation of specific DNA-binding proteins.

Choice E: Microarray analysis is similar to Southern and Northern blotting, but involves
hybridization of a large number of probes at once. The genomic DNA or cDNA being analyzed is
labeled with a fluorescent tag and placed on a gene chip containing complementary sequences for a
large number of genes. The degree or fluorescence corresponds to the mRNA expressed in the
particular sample.

Educational Objective:
Western blotting is used to identify proteins by first separating an unknown sample by gel
electrophoresis and subsequently probing that sample with antibodies specific for the protein of
interest.
Question 25

A 7-year-old African American male presents with fatigability, mild anemia, and spleenomegaly.
His hemoglobin electrophoretic findings are shown below. They reveal an abnormal hemoglobin
that migrates more slowly towards the anode as compared with, both a control, and a patient with
HbS disease.

The abnormal hemoglobin of this patient results from which of the following genetic alteration?

Answers

A. Trinucleotide expansion
B. Nonsense mutation
C. Missense mutation
D. Splice site mutation
E. Frameshift mutation

Explanation

Different proteins can be separated by the process of electrophoresis. Principle of this procedure is
that proteins move in an electrical field at different rates, depending on their molecular weight and
electric charge. Normally hemoglobin consists primarily of hemoglobin A (HbA), which migrates
rapidly toward the positive electrode. Hemoglobin electrophoresis is used for detecting presence of
abnormal hemoglobin in a patient suspected of having a hemoglobinopathy. In HbS disease, a
nonpolar amino acid (valine} replaces the negatively charged amino acid (glutamic acid} in both
the beta chains of the Hb molecule. This amino acid replacement decreases the polarity of the Hb
molecule and as a result HbS more slowly towards the anode. Similarly, in hemoglobin C disease
lysine is substituted for glutamate at the 6th position in the beta globin chain. Since lysine is a
positively charged amino acid, HbC mores even more slowly towards the anode. Hence the
abnormal hemoglobin in the patient discussed above is likely to be HbC. Hemoglobin C results
from a missense mutation. Missense mutation is a non-silent base pair substitution which produces a
change in a single amino acid (Choice C)

Choice A: Trinucleotide expansions increase the number of trinucleotide codes in the coding region
of a gene resulting in large and unstable proteins or alterations in the epigenetic effects of a
particular gene.

Choice B: Nonsense mutations introduce a stop codon (UAA, UAG or UGA) in mRNA. Because of
abnormally placed stop codon, premature termination of mRNA translation results in formation of
truncated proteins.

Choice D: Splice site mutations are those that occur at intron-exon boundaries. This alters the
spicing signal that is essential for proper excision of an intron.

Choice E: Frameshift mutations results from deletion or insertion of one or more base pairs. When
these extra base pairs are not a multiple of three, the effect produced is especially detrimental
because it alters all the downstream codons.

Educational Objective:
HbC is caused by a missense mutation that causes substitution of glutamate with lysine in the beta
globin chain, resulting in increased positive charge of the molecule. Thus, HbC moves more slowly
than both HbA and HbS on hemoglobin gel electrophoresis.
Question 26
A 14-year.old Caucasian girl presents to clinic for a routine checkup. She complains that her
menstrual cycles are irregular. Her menses last for seven to ten days. She had her menarche two
year ago and her last menstrual period was one week ago. Her past medical history reveals
appendectomy at the age of 4 years. Her BMI is 23kg/m2. Which of the following is the most likely
cause of this patient’s complaint?

Answers

A. Absent ovulation
B Complex atypical hyperplasia of the endometrium
C. Endometrial stimulation by progesterone

D. Increased FSH secretion


E. Endometrial atrophy

Explanation:

In most women of child-bearing age, menstrual cycles are regular and last for about 28 days with
menstrual flow duration ranging from 2 to 7 days. In the first 5 to 7 years after menarche and
in the last 10 years before menopause, variation in menstrual can be seen commonly. A predictable
and regular menstrual cycle indicates normal ovulation. If menstrual cycle varies markedly,
possibility of an anovulatory menses is high. Hence a possibility of an anovulatory cycle should be
considered in this patient. (Choice A)

Choice B: Complex atypical hyperplasia of the endometrium is associated with prolonged


unbalanced exposure to estrogen. This condition is most common in obese older women or in
women who receive unopposed estrogen during hormone replacement therapy.

Choice C: In a normal ovulatory cycle, estrogen in first half of the menstrual cycle is responsible
for proliferation of endometrial glands and in the latter part of the cycle, progesterone stimulates
secretory changes. All these alterations prepare the endometrium for the possible implantation of the
zygote. Sudden decline in progesterone secretion is responsible for menstrual bleeding. Therefore
endometrial stimulation by progesterone would not normally cause irregular or lengthy menses

Choice D: Normally, secretion of FSH from pituitary gland is regulated by estrogen and
progesterone secreted from corpus luteum of the ovary by a negative feedback mechanism. Absence
of this negative feedback, as is likely to occur in post-menopausal years, would lead to a state of
excess FSH secretion.

Choice E: Menstrual cycle ceases when the endometrium is no longer exposed to hormonal
stimulation. The end result is endometrial atrophy, as is seen routinely in post-menopausal women
Educational objectives
Anovulation is common during the first five to seven years after menarche and in the last ten years
before menopause. Usual clinical presentation of an anovulatory cycle is marked menstrual cycle
variability.
Question 27

A 42-years-old Caucasian female presents in the hospital complaining of a small amount of blood
stained discharge following sexual intercourse. Her menstrual history is normal and bleeding is
within normal limits. She is nulliparous. There is no associated abdominal pain or urinary
symptoms. She smokes two packs of cigarettes per day and consumes alcohol regularly.
Her family history reveals that her mother died at the age of 54 years due to carcinoma of breast. A
cervical biopsy was done which showed presence of cervical intra epithelial neoplasia grade III
(CIN III). Amongst the following which is strongest risk factors for this condition?

Answers

A. Low fiber diet

B. Smoking

C. Alcohol consumption

D. Infection with human papilloma virus (type 6 and 11)

E. Nulliparity

F. Multiple sexual partners

G. Early menarche

Explanation:

CIN III denotes severe dysplasia of lining cervical squamous epithelium. The strongest known risk
factor for development of CIN and invasive cervical carcinoma is infection of the cervical
epithelium with human papilloma virus (HPV) types 16 and 18. Other types implicated are type 31,
33 and 35). Type 6 and 11 HPV are associated more often with condyloma acuminatum of the
cervix rather than carcinoma. HPV is a sexually transmitted disease. Early age at first intercourse,
multiple sexual partners, and prostitution are considered risk factors for HPV infection. HPV DNA
is detected by hybridization techniques in over 95% of cervical cancer. A recently introduced
vaccine against HPV is believed to offer protection against this infection and development of
cervical cancer (Choice F)

Choice B: Smoking is an independent risk factor for cervical cancer. The risk of cervical carcinoma
in situ is about 4 to 5-fold higher in smokers compared to matched controls.

Choice C: Alcohol does not have any association with development of cervical cancer

Choice D: Type 6 and 11 HPV are associated with condyloma acuminatum of cervix.
Choice E: Nulliparity is associated with endometrial cancer. Multiparity is believed to be a
predisposing factor in cervical cancer.

Choice G: Early menarche has no role in development of cervical cancer.

Educational Objective:
Cervical epithelial infection with human papilloma virus (HPV), especially HPV 16 and HPV 18, is
the strongest known risk factor for development of CIN and invasive cervical carcinoma. Early age
at first intercourse and multiple sexual partners increase the incidence of HPV infection and
resultant cervical cancer.
Question 28

A 28-year-old Afro-American woman presents to the clinic in 30th week of pregnancy. She is a
primigravida. Her systemic examination shows pedal edema. She denies having abdominal pain or
vaginal discharge. No history of seizures is present. She eats a balanced diet and takes folic acid
supplements. Her blood pressure is 180/100 mmHg and her heart rate is 90/min. Which of the
following additional findings is most likely to be present in this patient?

Answers:

A. Splenomegaly

B. Proteinuria

C. Thrombocytosis

D. Ketonuria

E. Fasting hyperglycemia

Explanation:

Pre-eclampsia is the triad of hypertension, proteinuria and edema in pregnancy, although the
diagnosis can be made with just the first two criteria even if there is no edema present. Pre-
eclampsia occurs as a complication in about 7 to 10% of pregnancies, with onset typically after the
20th week of gestation. Some patients with preeclampsia develop complications like convulsions
when it is called as eclampsia. Patients with eclampsia develop DIC. This produces lesions in
kidney, liver, heart and brain. Preeclampsia is more common in primgravidas. It is thought that
abnormalities in placentation characterized by poor development of vessels leads to placental
ischemia. This produces endothelial injury with consequent release of release of vasoconstrictor and
thrombogenic substances. These substances are responsible for widespread thrombosis in arterioles
and capillaries of different organs. Normal pregnant women are resistant to the vasoconstrictor
effect of angiotensin. It is believed that in eclampsia there is loss of this resistance, which is
responsible for development of hypertension. (Choice B)

Choice A: Splenomegaly is a nonspecific finding that that is seen in many different disorders. Some
common causes include portal hypertension, myeloproliferative disorders and some infectious
diseases like malaria etc.

(Choice C) Thrombocytosis occurs most commonly in myeloproliferative diseases. Sometimes pre-


eclampsia may present with subclinical hepatic disease characterized by elevated liver enzymes,
hemolysis and thrombocytopenia (HELPP syndrome).
Choice D: Ketonuria occurs in starvation and diabetic ketoacidosis .Any condition with decreased
carbohydrate metabolism and increased fat degradation will produce ketosis.

Choice E: About 4% of pregnant women develop resistance to insulin during pregnancy, which
produces hyperglycemia. This is known as gestaional diabetes. And can result in abortion or
stillbirth. Women who develop gestational diabetes during pregnancy are at increased risk for
developing type II diabetes later in life

Educational Objective:

Preeclampsia is the triad of hypertension, proteinuria and edema during pregnancy. Eclampsia is
pre-eclampsia with presence of convulsions. Sometimes pre-eclampsia may present with subclinical
hepatic disease characterized by elevated liver enzymes, hemolysis and thrombocytopenia (HELPP
syndrome).
Question 29

A 42-year-old female presents to your clinic complaining of lower abdominal discomfort. She also
gives history of increased bleeding during her menses that last for 6 to 7 days. Examination shows
normal cervix but the uterus is found to be enlarged. A biopsy reveals presence of normal appearing
endometrial glands surrounded by some endometrial stroma within the myometrium. The most
likely diagnosis is

Answers:

A. Leiomyoma

B. Endometrial carcinoma

C. Adenomyosis

D. Dysfunction uterine bleeding

E. Atypical endometrial hyperplasia

Explanation:
Adenomyosis is the presence of islands of endometrial glands and stromal tissue within deep
myometrium. Presence of endometrial tissue outside the uterus is called endometriosis. Though this
condition is relatively common, its etiology is unknown. It is proposed that it represents a
metaplastic change in the remnant of the mullerian system. Adenomyosis typically, affects middle
aged parous women. It is rare in post-menopausal women except as a complication of tamoxifen
administration. Symptoms include pelvic pain, menorrhagia and dysmenorrhea. Physical
examination classically reveals a uniformly enlarged uterus. Uterine enlargement results primarily
from uterine smooth muscle hypertrophy and hyperplasia in response to ectopic endometrial
glandular tissue. (Choice C)

Choice A: Leiomyoma is a benign tumor of smooth muscle cells of uterus. Physical examination
shows an irregularly enlarged uterus as opposed to adenomyosis where the uterine enlargement is
globular.

Choice B: Most common form of endometrial carcinoma is an adenocarcinoma. Hence this


condition will show infiltrating neoplastic glands in the myometrium. The most common
presentation of endometrial carcinoma is uterine bleeding in a postmenopausal woman.

Choice D: Dysfunctional uterine bleeding (DUB) refers to uterine bleeding not associated with an
organic cause in a woman of child bearing age. It is usually caused by imbalanced hormonal
stimulation of the endometrium.

Choice E: Unremitting and unopposed endometrial stimulation by estrogen results in endometrial


hyperplasia. When histological appearance of hyperplastic glands approaches the appearance of
adenocarcinoma but falls short of fulfilling all the criteria of malignancy, then it is called as atypical
endometrial hyperplasia. Patients will present with menorrhagia.

Educational Objective:
A uniformly enlarged uterus with normal appearing endometrial tissue within the myometrium
indicates a diagnosis of adenomyosis. It usually presents as menorrhagia and dysmenorrheal.
Question 30
A 60-year-old female presents to the hospital with complaints of lower abdominal discomfort,
urinary frequency and abdominal enlargement. Physical examination reveals an adnexal mass on the
right side. This patient is most likely to have elevated levels of which of the following?

Answers

A. CA-125
B Carcinoembyonic antigen
C. CA 19.9
D. α- fetoprotein
E. ß – HCG
F. DHEA

Explanation:

This patient is most likely to be suffering from ovarian cancer; hence she is likely to have raised
CA-125 levels in her serum. CA-125 is produced by malignant ovarian epithelial tumors. Between
80 to 85% of females with epithelial ovarian cancers have a CA-125 level greater than 35 U/ml.
However about 50% of patients with early stage I or II disease (who are more amenable to
treatment), do not show significant rise of CA-125 levels. Since levels of this glycoprotein can also
be raised in many non-neoplastic disorders like endometriosis, pelvic inflammatory disease and
uterine fibroids, its estimation as a screening tool carries high false positivity rates. However in
post-menopausal women with adnexal mass and CA-125 levels greater than 65 U/ml, the test has a
specificity of around 80% and sensitivity of about 95%. Estimation of CA-125 levels is also useful
for appraisal of therapeutic response during treatment of these patients (choice A).

Choice B: CEA is a serum marker that is produced by about 70% of colorectal and pancreatic
cancers. Not useful in ovarian cancers.

Choice C: CA-19.9 is a carbohydrate antigen that is elevated in about 74% of patients with
pancreatic cancer, in about 40% of patients with hepatic cancer, in 42% of patients with gastric
cancer and in about 20% of patients with colon cancer,

Choice D: Serum alpha-fetoprotein levels may be elevated in patients with hepatocellular carcinoma
and nonseminomatous germ cell tumors of the testis (e.g. yolk sac tumors). Its levels rise in many
non-malignant disorders also.

Choice E: ß-HCG is produced used for diagnosis and follow up of patients with tumors of placenta
like hydatidiform mole and choriocarcinoma.

Choice F: DHEA is produced by androgenic zone of adrenal cortex. Its levels may be elevated in
patients with excess production of adrenal androgens, as in congenital adrenal hyperplasia adrenal
cortical neoplasms.
Educational Objective:
Ovarian malignancy is the most likely explanation for an adnexal mass in an elderly female.
Approximately 90% of ovarian malignancies originate from the ovary's surface epithelium. CA-125
is produced by malignant ovarian epithelial tumors and can be used as a serum marker for this
condition. However consistent rise of CA-125 is seen only in advanced stages of ovarian
malignancies of epithelial origin. Hence its role as a screening procedure for early detection of
ovarian cancers is very limited.
Question 31
A 30-year-old female presents in the clinic with a history of amenorrhea and nipple discharge. She
is unmarried and sexually inactive. She eats a balanced diet and exercises regularly. Her breast
examination is unremarkable except for the presence of nipple discharge. She does not take any
medications. Her mother died of breast cancer at age 50. The most likely diagnosis in this patient is:

Answers

A Intraductal papilloma

B. Fibroadenoma

C. Lobular breast carcinoma

D. Fibrocystic disease of the breast

F Pituitary adenoma.

G. Paget’s disease of nipple

Explanation

The history in this patient is suggestive of increased secretion of prolaction. Of the choices
listed above, increased prolactin secretion would be seen in a pituitary tumor. Benign tumor of
lactotrophs of pituitary gland is not unusual with an annual incidence of 3 cases per 100,000
persons. Hyperprolactinemia causes galactorrhea and amenorrhea in females. Galactorrhea
results from the direct stimulatory affect of prolactin on the mammary glands, and
amenorrhea results from the inhibitory effect of prolactin on GnRH secretion. Causes other than
pituitary adenoma that can produce hyperprolactinemia are: damage to the pituitary
hypothalamic axis, systemic disorders like cirrhosis of the liver and hypothyroidism and
medications like dopamine receptor blockers, amitryptaline, fluoxetine and verapmil. (Choice F)

Choice A: Intraductal papilloma is a benign tumor of small or the large ducts of the breast. It is
the most common cause of spontaneous nipple discharge. However, in intraductal papilloma, the
discharge is usually bloody or serosanguinous.

Choice B: Fibroadenoma is a benign tumor of the breast that occurs most often in young
females. It presents as a freely movable mass in the breast.

Choice C: Lobular breast carcinoma typically presents as breast lump. Nipple discharge may be
seen, but amenorrhea is not a feature of this disease. The diagnosis rests on histological
examination.

Choice D: Fibrocystic disease of the breast is seen most commonly in the age group of 25 to 45
years. The exact pathogenesis is unknown, but hormones obviously play a role, as the disease
waxes and wanes during the menstrual cycle. The process is often bilateral and the breasts feel
lumpy and dense.

Choice E: Paget’s disease presents as a crusted lesion of the nipple and may be associated with a
sersanguinous discharge. Clinically it presents as eczema - like lesion of the nipple and the
surrounding skin. Underlying carcinoma of the breast is seen in majority of cases.

Educational Objective:

Prolactinomas are the most common pituitary adenomas. Excess prolactin produced by these
lesions can cause infertility galactorrhea and amenorrhea in females. In men prolactinoma
presents with impotence, loss of libido and infertility. Visual changes may also occur due to
compression of the optic chiasma.
Question 32
A 28-year-old female presents to the clinic complaining of foul smelling vaginal discharge. Wet
mount preparation of the discharge shows many leukocytes. Application of KOH solution to the
discharge yields a strong fishy odor. The most likely diagnosis is

Answers:
A. Candidiasis

B. Chlamydia infection

C. Atrophic vaginits

D. Gonorrheal infection

E. Trichomonas infection

F. Bacterial vaginosis

Explanation:

History and examination in this patient are strongly suggestive of bacterial vaginosis (BV), a
common vaginal disease caused most commonly by the bacillus, Gardenella vaginalis. Other
microorganisms implicated are Mycoplasma and anaerobes like Mobiluncus, Prevotella and
Peptostreptococcus. Risk factors for BV include multiple sexual partners. Loss of hydrogen
peroxide producing lactobacilli is a contributing factor. Addition of 10% potassium hydroxide
(KOH) to a sample of vaginal discharge accentuates the fishy odor (caused due to production of
volatile amines, trimethylamine). Other diagnostic criteria of BV are: (1) presence of clue cells in
vaginal discharge. These are epithelial cells covered by numerous coccobacilli that impart a
granular appearance to the epithelial cells (2) Vaginal discharge has a ph > 4.5. Microscopic
examination of vaginal discharge shows reduced number of gram positive rods (lactobacilli) and
increased number of gram variable and gram negative rods – Nugent test. (Choice F)

Choice A: Candida albicans is the most common cause of fungal vaginitis. Patients present with
intense vaginal pruritus, white curd-like discharge and labial erythema. KOH examination of the
vaginal discharge shows yeast forms and pseudohyphea. No abnormal odor is elicited with the
addition of KOH.

Choices B and D: N gonorrhoea and C. trichomatis are sexually transmitted diseases that commonly
exist as co-infection. In women these organisms typically cause cervicitis accompanied by a
purulent white discharge. Infection can progress to pelvic inflammatory disease. Gram stain of
vaginal discharge may reveal N. gonorrhea but C. trichomatis cannot be visualized in this manner.

Choice C: Atrophic vaginitis occurs in postmenopausal and hypoestrogenemic women. Patients


complain of vaginal dryness, serosanguinous or watery discharge and dyspareunia.
Choice E: Trichomanas is a flagellated protozoan that also causes vaginitis. Patients develop a
yellow-green foamy foul smelling discharge. Motile trophozoites with flagellae are seen on wet
mount microscopy.

Educational objective
Enhancement of fishy odor on addition of potassium hydroxide (KOH) to vaginal discharge is a
sign of bacterial vaginosis. Presence of ‘Clue cells’ on saline wet mount microscopy also suggest
this diagnosis.
Question 33

A 45 -year-old Caucasian female presents to your office with a palpable nodule in the left breast.
Needle biopsy of the mass reveals that larger ducts are distended by islands of pleomorphic cells.
These islands show central necrosis. Mitotic count is: 6 mitotic figures/10HPF. The lesion does not
extend beyond the ductal basal membrane. Calcification is seen. Which of the following is the most
likely diagnosis in this patient?

Answers:

A. Paget’s disease

B. Sclerosing adenosis

C. Medullary carcinoma

D. Comedocarcinoma

E. Lobular carcinoma in situ

F. Phylloides tumor.

Explanation:

The histological picture described above is typical of comedocarcinoma of the breast.


Comedocarcinoma is a subtype of ductal carcinoma in situ (DCIS). Other variants of DCIS are:
solid, cribriform, papillary, and micropapillary. Ductal carcinoma in situ (DCIS) in its pure form is a
preinvasive cancer of the breast. DCIS represents malignant clonal proliferation of cells that have
not breached the surrounding basement membrane. Usually only a single ductal system appears to
have DCIS. However in DCIS, far more extensive lesions can occur with widespread involvement
of the breast parenchyma. In addition to intraductal proliferation of neoplastic cells, DCIS show
chronic inflammation and periductal concentric fibrosis. Eventually the necrotic areas within ducts
undergo calcification, which can be detected on mammography. When comedocarcinoma becomes
extensive, the lesion may be palpable as a poorly defined nodule. In mammographically screened
populations, DCIS now represent 15-30% of all breast cancers. 50% of carcinomas identified on
mammography turn out to be DCIS (Choice D)

Choice A: Paget,s disease of the nipple is an eczema like lesion of the nipple and the surrounding
skin. Nipples show a weepy and crusted appearance. In majority of cases there is an underlying
cancer. Sometimes, when DCIS becomes extensive, it may present as Paget’s disease.

Choice B: Sclerosing adenosis can be easily misdiagnosed as cancer. Lesions show proliferating
tubules that are elongated and compressed. Stroma is dense. Myoepithelial cells are present around
the proliferating tubules. It represents a form of adnosis and is not a cancer.
Choice C: Medullary carcinoma is an invasive carcinoma of the breast that is often misdiagnosed
clinically as fibroadenoma. It is characterized by solid sheets of large pleomorphic cells, without
gland formation and having pushing rather than infiltrating borders. Mitotic activity can be brisk.
Significant lymphoplasmacytic infiltrate is seen around and within the tumor. Prognosis of this form
of breast cancer is better than conventional invasive ductal cancer with 10 year survival rates
approaching 80%.

Choice E: Lobular carcinoma is a multicentric form of breast cancer that can be bilateral in up to 30
to 40% of cases. Often it is detected incidentally in breasts removed for other reasons.
Microscopically, there is lobular distension produced by proliferation of uniform, small round cells.
20 to 30% cases become invasive cancers.

Choice F: Clinically phylloides tumor appears similar to fibroadenoma, but has increased
cytological atypia and stromal cell overgrowth .The resultant architecture is described as leaf like.

Educational Objective:

Comedocarcinoma (DCIS) is an in situ ductal carcinoma characterized by proliferation of solid


sheets of pleomorphic cells with central necrosis within the ductal system of the breast. In its pure
form its prognosis is good. However, if left untreated they often evolve into invasive cancers.
Question 34
A 32-year-old female complaints of excessive uterine bleeding occurring at frequent intervals. A
urine pregnancy test is negative. Pelvic ultrasound reveals endometrial hyperplasia and a left-sided
adnexal mass. In this patient, the adnexal mass is most likely to be a:

A. Dysgerrninoma

B. Mucinous cystadenocarcinoma

C. Benign cystic teratoma

D. Sertoli cell tumor

E. Granulosa cell tumor

Explanation:

This is having endometrial hyperplasia, which indicates existence of a hyperestrogenemic state.


Since an adnexal mass is also present, it would be logical to correlate hyperestrogenemia with this
mass. An ovarian tumor that is commonly seen to produce estrogen is a granulosa cell tumor (GCT).
About 75% of granulosa cell tumors are associated with hyperestrinism. Excess estrogen produced
by such tumors has been linked to precocious puberty, endometrial hyperpiesia and endometrial
carcinoma. Majority of GCT occur in the older age group. Some cases ar seen in first two decades
of like where they produce isosexual precocity. Histopathologically, this tumor consists of cells with
typical coffee-bean nuclei. In addition to this typical nuclear morphology, there is presence of Call-
Exner bodies that are small follicle-like structures filled with eosinophilic secretions (Choice E).

Choice A: Dysgerminoma is a rare malignant ovarian tumor of germ cell origin. It comprises less
than 1% of all ovarian tumors and it does not secrete estrogen. The tumor is most common in
younger age group of 20 to 30 years It is the ovarian equivalent of the testicular seminoma.

Choice B: Mucinous cystadenocarcinoma is a malignant neoplasm


onginating horn the ovarian surface (c elomic_ germi n al) epithelium
These nimors produce mucin, not estrogen. Mutinous cystadenocarcinoma
may progress to pseuclornwoma pentonei, a condition ober:, the
peritoneal caMtyfills moth mucin due to widespread intraperitoneal
metastases.

Choice C: Benign cystic teratoma (dermoid cysts) comprises about 20% of all ovarian tumors. A
variety of tissues like skin, dermal appendages, cartilage, bone, teeth and many others are often
present in these tumors. Dermoid cysts do not secrete estrogen but may produce excessive amounts
of thyroid hormone if composed primarily of thyroid tissue, a condition known as struma ovarii

Choice D: Sertoli cell tumor: these are uncommon ovarian tumors comprising less than 0.1% of all
ovarian tumors. They are very often androgenic, hence the older designation, androblastoma

Educational objective:
Granulosa cell tumors are estrogen-secreting ovarian tumors. The hyperestrogenemic state can
cause endometrial hyperplasia and abnormal uterine bleeding. It also predisposes to development of
endometrial carcinoma.
Question 35

A 24year-old Caucasian male is admitted to the hospital with a one-week history of cough and chest
pain. Lung auscultation reveals crackles over the right lower lung lobe. When placed in an
upright glass tube, his anticoagulated blood cells fall at a rate of 35 mm per hour. This finding is
most likely related to the action of which of the following substances?

A. Thromboxane

B. Leukotriene

C. Prostaglandins

D. Interleukin-6 (IL-6)

E. Platelet activating factor

F. Bradykinin

Explanation:

During an inflammatory response, neutrophils and macrophages release certain cytokines like tumor
necrosis factor-alpha (TNF-α), IL-1 and IL-6. These cytokines mediate systemic inflammatory
response (SIRS), and also stimulate hepatic production of acute phase proteins like fibrinogen,
ferritin, C-reactive protein, serum amyloid A, serum amyloid P and many others. One of the acute-
phase proteins, fibrinogen, cause erythrocytes to stack over each other (rouleaux). These stacks of
RBC, sediment at a faster rate compared to individual erythrocytes. The rate at which erythrocytes
sediment every hour, is called, erythrocyte sedimentation rate, or ESR. It is a non-specific marker of
ongoing inflammation in the body (Choice D).

Choice A: Thromboxane A2 is one of the products of the cyclooxygenase pathway. It is synthesized


by platelets and produces vasoconstriction and platelet aggregation.

Choice B: Leukotrienes are metabolites of arachidonic acid. Many different forms exist. Their
produce chemotaxis, vasoconstriction, vasospasm and increased vascular permeability.

Choice C: Apart from thromboxane, other prostaglandins produced during inflammation are mainly
PGE2, PGD2, PGF2α, and PGI2. They have varied functions like inhibition of platelet aggregation,
vasodilatation, pain and fever. PGE2 is a very potent hyperalgesic.
Choice E: Platelet-activating factor (PAF) is secreted by leukocytes and mast cells at the site of
inflammation.. Its main effects are bronchoconstriction and priming of leukocytes. It is much more
potent than histamine in producing these effects.

Choice F: Bradykinin is a component of the kinin system. It causes vasodilatation, increases


vascular permeability, stimulates smooth muscle contraction, and mediates pain.

Educational Objective:

Three cytokines, IL-1, IL-6, and TNF-α mediate systemic inflammatory response (SIRS) and
stimulate hepatic secretion of a number of acute-phase proteins High levels of one of these –
fibrinogen increases the erythrocyte sedimentation rate (ESR) by promoting rouleux formation.
ESR is a nonspecific marker of inflammation
Question 36

A 46-year-old women had a left adnexal mass, which was resected. On gross examination, cut
surface of the mass is yellow in color. Histological diagnosis of this lesion is granulosa-theca cell
tumor. Which of the following substances this tumor is most likely to secrete?

Answers

A Cortisol

B Estrogen.

C. CA-125

D. Alpha-1 fetoprotein (AFP

E. Androgen

F Human chorionic gonadotropin (HCG)

Explanation:

Granulosa-theca cell tumors comprise approximately 5% of all ovarian tumors. They are seen
primarily in the child- bearing age and sometime in postmenopausal women. In small number of
cases, they can be seen in young adolescent girls (juvenile granulosa cell tumor). Derived from the
ovarian stroma, these tumors fall under the broad category of sex cord- stromal tumors. Typically,
granulosa-theca cell tumors are unilateral and of and variable size. There lipid content (and
therefore their hormonal activity) manifests as yellow coloration of the cut surface (luteinized
granulosa cells).. Granulosa-theca cell tumors tend to contain a predominance of granulosa cells and
a scattering of theca cells. Because the granulosa and theca cells secrete estrogen variants, these
tumors have a feminizing effect. Very few are androgenic. If large quantities of estrogen are
released, precocious sexual development can occur in prepubertal girls. Older women may develop
fibrocystic change of the breast, endometrial hyperplasia, or endometrial carcinoma. Histologically,
granulosa cells are small and cuboidal cells with characteristic coffee bean nuclei. They grow in
cords or sheets. Small, gland-like structures that contain an acidophilic material may be present.
These are termed Call-Exner bodies and are suggestive of immature follicles. Thecoma cells present
as clusters or sheets of spindle shaped cells. Tumors that primarily consist of theca cells are almost
always benign in nature (Choice B)

Choice A: Cortisol levels may be abnormal in patients with hypothalamic, pituitary, or adrenal
malfunction. Measurement of cortisol levels is usually not helpful in the evaluation of ovarian
tumors
Choice C: Cancer antigen-125 {CA-125) is found in elevated amounts in malignant surface
epithelial cell tumors of the ovary. This glycoprotein is best used in monitoring therapeutic
response. CA-125 is not an appropriate screening tool for ovarian cancer because of it low
sensitivity and specificity. It can be raised neoplastic disorders of many other organs and also in
certain non-neoplastic conditions like endometriosis and pregnancy.

Choice D: Alpha-fetoprotein {AFP) is normally produced by the fetal liver cells and by the yolk sac
of the embryo. It levels are raised in several neoplastic and neoplastic disorders. Its utility is for
diagnosis and follow- up of patients with hepatocellular and germ cell tumors of ovary and testis.

Choice E: Rarely granulosa cell tumors can produce androgens and have a masculinizing effect

Choice F: HCG levels are raised in trophoblastic and/ or germ cell tumors, including extra gonadal
trophoblastic and/ or germ cell tumors. Estimations of HCG are also used for confirmation of
pregnancy.

Educational Objective:

Granulosa cell tumors are sex-cord stromal tumors of the ovary that secrete estrogen and can
produce endometrial hyperplasia or endometrial carcinoma women of child bearing age group. It
can also cause isosexual precocious puberty in young girls.
Question 37

A 5-year-old - boy present to your office with complaints of passing dark colored urine for last three
days. He also gives past history of bloody diarrhea along with colicky abdominal pain. Physical
examination reveals palpable skin lesions on his legs that do not blanch on pressure. What
additional findings are likely to be present in this patient?

A Joint pain

B. Impetigo

C. Oral ulcers

D. Red eyes

F Lymphadenopathy

Explanation:

Clinical presentation in this child is consistent with the diagnosis of Henoch-Schonlein purpura
(HSP), the most common small vessel vasculitis (an IgA mediated leukocytoclastic vasculitis) in
the pediatric age group.. The disease typically affects boys 2-10 years old (75% cases occur in this
age group) and is often preceded by oral and upper respiratory tract streptococcal infection.
Symptoms of HSP generally develop a few weeks after the associated illness resolves. It is believed
to be an immune complex disorder. HSP most often resolves spontaneously. Rarely, it may produce
death due to renal involvement. Following organ systems are affected most often:
1. Gastrointestinal tract: Intermittent severe abdominal pain is common in HSP. Vasculitis within the
GI tract may result in upper and lower GI bleeding as well as bowel wall edema. Patients with HSP
also have an increased risk of intussusception.
2. Kidneys: Renal involvement in HSP is seen in about 70% of the cases. They may range from
microscopic hematuria to chronic renal failure. Some patients may develop rapidly progressive
crescentic glomerulonephritis. Renal manifestations of HSP are more severe in adults. IF
-microscopy shows IgA deposition in glomerular mesangium.
3. Skin: HSP classically causes "palpable purpura" on the buttocks and on extensor surface of the
lower extremities. The cutaneous findings in HSP result from leukocytoclastic vasculitis.
4. Joints: Self-limited migratory arthralgias without frank arthritis are seen most commonly in the
large joints of the lower extremity (Choice A)

Choice B: Impetigo is a superficial skin infection caused by streptococcus and staphylococcus.


Choices C: Oral ulcers are a non specific finding and can be a component of many illnesses, both
immune mediated and non immune mediated.

Choice D and E: Swollen lymph nodes and red eyes are characteristic findings in Kawasaki disease.
Other symptoms of Kawasaki disease include high fever, strawberry tongue, perioral erythema and
fissuring, and periungual desquamation. It may produce aortic aneurysm in up to 25% of the cases

Educational Objective:

Honoch-Schonlein purpura (HSP) generally affects young children and is classically preceded by an
upper respiratory infection. This IgA-mediated immune complex hypersensitivity disorder
commonly causes abdominal colics, joint pain, and palpable purpura in lower extremity and
buttocks. Renal involvement is frequent and rarely it may result in death of the patient.
Question 38

A 36-year-old male presents with hematuria and hypertension. The immunofluorescent (IF) findings
of his kidney biopsy, using anti IgG, are shown below. What is the likely clinical diagnosis and what
would be the light microscopic findings in this case?

Answers

A Normal glomerular appearance

B. Capillary wall thickening

C. Focal glomerular sclerosis

D. Crescentic glomerulonephritis

E. Basement membrane splitting

Explanation:
On immunofluorescence, linear deposits of immunoglobulin G are seen along the glomerular
basement membrane. This finding indicates that the implicated antigen is present on the glomerular
basement membrane. These changes are characteristic of Goodpasture syndrome. The syndrome
consists of crescentic glomerulonephritis along with pulmonary manifestations. The antigen against
which anti-GBM antibodies are localizing consists of carboxy terminus of the NCI domain of the α3
chain of type IV collagen. This type of collagen is part of the glomerular basement membrane.
Characteristic renal biopsy findings in Goodpasture syndrome are presence of necrotizing
glomerulonephritis with crescent formation. Anti-GBM antibodies cross-react with pulmonary
alveolar basement membrane also and produce pulmonary hemorrhages. Anti-GBM disease can
complicate other disorders like membranous glomerulonephritis and Alport syndrome (Choice D).

Choice A: Normal glomeruli are found on light microscopy in minimal change disease (MCD), a
condition affecting mostly children that presents as a nephrotic syndrome. There are no deposits on
immunofluorescence.

Choice B: Uniform, diffuse capillary wall thickening on light microscopy is a sign of membranous
glomerulopathy. The typical presentation is nephrotic syndrome (generalized edema, marked
proteinuria, hypoalbuminemia, hyperlipidernia and lipiduria). Immunofluorescence demonstrates
granular deposits of IgG and C3 along the glomerular basement membrane.

Choice C: In focal segmental glomerulosclerosis (FSGS) IgM and C3 deposits are found in the
sclerotic areas of the glomeruli on immunofluorescence. Linear deposition of immunoglobulins is
not typically found.

Choice E: Basement membrane splitting of glomerular capillaries along with capillary wall
thickening is seen in Alport syndrome and in membranoproliferative glomerulonephritis (MPGN)
type 1. In type I MPGN, granular deposits are seen along basement membrane by
immunofluorescence. On light microscopy, the glomeruli of MPGN have a lobular appearance.
There is increased mesangial matrix along with proliferation of mesangial cells. On silver staining
characteristic ‘tram track appearance’ of basement membranes is seen. Most cases of MPGN type I
are idiopathic but it can be secondary to infections (e.g. hepatitis B and C, malaria), systemic
autoimmune diseases (e.g. SLE, rheumatoid arthritis), neoplastic disorders, and hereditary diseases.
In Alport syndrome, symptoms of nephritis are accompanied by deafness.

Educational objective:
Anti-GBM antibodies react with the α3 chain of collagen type IV that is normally present in the
glomerular basement membrane. Anti-GBM antibodies are found in Goodpasture syndrome, a
condition characterized by pulmonary hemorrhages and rapidly progressive glomerulonephritis
(RPGN). On light microscopic examination, there is necrotizing glomerulonephritis along with
crescent formation. On immunofluorescence, linear deposits of IgG end C3 along the glomerular
basement membrane are seen.
Question 39

A 21-year-old Japanese American male presents to the hospital with dark dusky colored urine.
Recently he had flu-like symptoms. On examination he is having purpuric skin lesions that are more
numerous on the trunk. Skin biopsy reveals granular deposits of IgA in the dermal capillaries. This
patient is likely to be suffering from:
Answers
Α. Post-infectious glomerulonephritis

B. ANCA-associated rapidly progressive glomerulonephritis

C. Berger’s disease

D. Membranous glomerulopathy

E. Membranoproliferative glomerulonephritis

Explanation:

The patient is likely to be suffering from IgA nephropathy, also called Berger’s disease. It was first
describe by Berger and Hinglais in 1968. Worldwide it is the commonest primary
glomerulonephritis, affecting about 10% of the patients having end stage chronic renal disease. It is
more common in children and young adults. Patients usually present with painless microscopic or
macroscopic hematuria, 2-3 days after an upper respiratory tract infection. The hematuria typically
lasts for several days and then subsides temporarily, retuning every few months. Proteinuria also
may be seen in many patients. Further course of this disease is variable. 5 to 10% develop nephritic
syndrome. About 25 to 40% go on to develop chronic renal failure. Recurrence may be seen in the
transplanted kidney also. On light microscopy there are no specific findings. Glomeruli may appear
normal or show different lesions like necrotizing glomerulonephritis, focal sclerosis or crescentic
glomerulonephritis. Diagnosis is by immunofluorescent microscopy, which shows prominent IgA
deposits in the mesangium. When IgA nephropathy is associated with extra-renal symptoms, it is
considered as a component of Henoch-Schonlein disease. Purpuric lesions can be seen on extensor
surfaces of arms, legs and buttocks. When the purpuric skin lesions are biopsied, light microscopy
reveals a necrotizing vasculitis of the small dermal vessels along with areas of subepidermal
hemorrhages. Immunofluorescence microscopy shows IgA deposition in dermal capillaries. In
some cases, IgA deposition in glomeruli may be secondary to diseases like chronic liver disease,
HIV infection, Crohn’s disease etc (Choice C)

Choice A: Post infectious glomerulonephritis usually occurs in the pediatric age group. Typically it
occurs 1 to 4 weeks after infection with certain nephritogenic strains of streptococci.. Clinically it
presents as abrupt onset of hematuria, edema, proteinuria, hypertension and altered renal function.
On light microscopy, glomeruli are enlarge and hypercellular. IF microscopy shows glomerular
deposits of IgG and C3. It is an immune complex disorder.
Choice B: Also called as pauci-immune crescentic glomerulonephritis. It is characterized by
absence or minimal staining for immunoglobulins by IF microscopy. This picture is seen in about 40
to 50% of patients with crescentic glomerulonephritis (rapidly progressive glomerulonephritis). It
may be a component of certain systemic vasculitic syndromes. In such cases, anti-neutrophil
cytoplasmic antibodies (ANCA) are present and serve as a marker for this disease. Important ANCA
associated glomerulonephritis are: idiopathic crescentic glomerulonephritis, microscopic
polyarteritis nodosa and Wegener’s granulomatosis.

Choice D: Membranous glomerulopathy may be primary or secondary. Primary membranous


glomerulonephritis is usually a disease of adulthood and is characterized by peripheral granular
deposition of IgG and C3 in glomerular capillary walls. It usually presents as nephrotic syndrome.

Choice E: In MPGN, granular deposits of C3 and IgG are seen along the basement membrane by
immunofluorescence microscopy. On light microscopy, the glomeruli of MPGN have a lobular
appearance. There is increased mesangial matrix along with proliferation of mesangial cells.

Educational Objective:
Painless hematuria 2-3 days after an upper respirator/tract infection is suggestive of Ig.A
nephropathy (Berger’s disease). Diagnosis is made by the detection of IgA deposits in the
glomerular mesangium on immunofluorescence microscopy. When IgA nephropathy is
accompanied by extra renal symptoms, it represents a component of Henoch- Schonlein purpura
Question 40

A 42-year-old female was operated for invasive carcinoma of the uterine cervix. After few days she
starts experiencing dull aching pain on right side of the abdomen. On physical examination, a deep
seated mass is felt in the right upper abdominal quadrant. Which of the following is the most likely
possibility that can explain this pain?
Answers

A Renal cell carcinoma


B Interstitial nephritis

C. Renal cystic disease

D. Renal artery thrombosis

E. Hydronephrosis

F Vesico-uretral reflux

Explanation:

A radical hysterectomy with patio- reconstruction surgery was probably carried out in this patient.
This surgery is elaborate and technically difficult as numerous important anatomic structures lie in
close proximity within the pelvis. For example, the ureters courses just posterior to the uterine
artery near the lateral fornix of the vagina and can easily be transected or ligated during
manipulation of the uterine artery. When ureter is ligated accidentally, the kidney continues to
produce urine, but the passage for drainage of urine is blocked. The end result is distention of the
ureters and the calyceal system of the kidney. If the obstructive pathology persists, it leads to
progressive atrophy of the renal parenchyma and the glomerular filtration rate begins to diminish.
Obstruction to urinary outflow is also associated with onset of interstitial inflammation. This
produces progressive fibrosis in the kidney. However with sudden and complete obstruction, as
would occur with ligation of the ureter, glomerular filtration stops within a short time and renal
enlargement is mild. Renal enlargement is marked only when the blockage in intermittent and
incomplete as occurs with renal stones. In these cases glomerular filtration is not suppressed. Pain
in acute obstruction is attributed to distension of the collecting system or renal capsule. (Choice E).

Choice A: Renal cell carcinoma produces painless hematuria, flank pain and a palpable mass.
These signs and symptoms are unlikely to be precipitated solely by a pelvic surgery.

Choice B: Interstitial nephritis is classically medication-induced. Signs and symptoms include fever,
arthralgias and transient rash.
Choice C: Adult polycystic kidney disease is an autosomal dominant condition characterized by
multiple renal, pancreatic and hepatic cysts. This disease typically produces gradual onset of
chronic renal failure

Choice D: Post-surgical patients are at increased risk for development of deep vein thrombosis due
to inactivity and hypercoagulability of blood. However, renal artery thrombosis is rare

Choice F: Vesicoureteral reflux can be a complication of prostatectony or bladder surgery. Due to


backflow of urine, it predisposes to pyelonephritis and hydroureteronephrosis

Educational Objective:

The ureters are at risk of injury during pelvic surgeries. Hysterectomies are especially risky because
the ureters course just posterior to the uterine arteries and can be accidentally ligated during this
procedure. Resultant symptoms are produced due to distension of renal calyceal system (diminished
GFR of the affected kidney, flank pain) and renal capsule (pain).
Question 41

There has been an outbreak of streptococcal skin infection in a small community. 2 to 3 weeks later
several members of this community present to the hospital with dark cola colored urine and facial
edema. Of the following, which factor is going to have maximum affect on the prognosis of these
patients?

Answers:

A. Anti-streptolysin O titre (ASO titer)

B. Age

C. Prompt steroid treatment

D. Family history

E. Race

F. Body mass index

Explanation:

The classic presentation of post-streptococcal glomerulonephritis is a young child developing


malaise, fever, nausea and cola-colored urine, 2-3 weeks after recovering from a streptococcal skin
or-throat infection. Bacteria implicated most commonly are groupA ß-hemolytic streptococci, M
types 1, 2, 4, 12, 18, and 25. Although this diseases follows bacterial infection, its pathogenesis is
immune-mediated and is not due to direct infection by microorganisms. Urine shows presence of
dysmorphic red blood cells, RBC casts, and mild proteinuna (<1 g/day). Periorbital and facial
edema and hypertension are also usually detected. Therapy includes loop diuretics and vasodilators
to relieve the edema and hypertension.
Age is the most important prognostic factor in patients with post-streptococcal glomerulonephritis.
95% of children recover completely with conservative therapy. 1-2 % of the patients develop
chronic glomerulonephritis and less than 1% progress to RPGN. The prognosis in adults is not as
good. Only 60% of sporadic cases in adults will resolve completely: the rest will go on to develop
chronic glomerulonephritis or rapidly proliferative glomerulonephritis. In adults complete recovery
is less likely when the initial episode is associated with severe renal impairment, persistent
proteinuria or nephrotic syndrome. Progression is more common in patients who develop crescentic
glomerulonephritis. (Choice B)
Choices D, E and F: A patient's race, BMI, and family history have no bearing on the prognosis in
post-streptococcal glomerulonephritis.

Choice A: Elevated anti-streptolysin 0 titers are diagnostic of a recent streptococcal infection.


Raised levels are found in > 90% of the patients. ASO titers begin to rise after 1 week of primary
infection and return to normal after 3 to 4 months. Elevated levels help confirm the diagnosis of
post-streptococcal glomerulonephritis, but do have any influence on the prognosis.

Choice C: Treatment consists of eliminating streptococcal infection and providing supportive


therapy. In addition to these, diuretics and antihypertensive are also employed. Steroids are usually
employed in membranous or minimal change glomerulopathy.

Educational Objective:

Age is an important prognostic factor in post-streptococcal glomerulonephritis. 95% of affected


children, but only 60% of affected adults recover completely.
Question 42
A 40- year-old homeless man is brought to the ER. He is having persistent vomiting and prolonged
agitation. His breath odor is faintly sweet. He is also oliguric. After initial evaluation, a renal biopsy
is done that reveals marked ballooning and vacuolar degeneration of proximal renal tubules.
Multiple crystalline structures are also seen in the lumen of the tubules that are identified as oxalate.
Glomeruli are normal looking. His condition is likely to be caused by?

Answers:

A Prolonged hypotension

B Liver failure

C. Toxic renal injury

D. Chronic malnutrition

E Septicemia

Explanation:

The patient is having oliguria and his renal biopsy is indicative of toxic acute tubular necrosis
(ATN). Presence of oxalate crystals in renal tubules in a setting of ATN is indicative of ethylene
glycol poisoning. Ethylene glycol is a widely used in automobile antifreeze engine coolants,
hydraulic brake fluids and as a solvent for paint and plastics. It is consumed as an intoxicant as its
effects are similar to ethyl alcohol. Ethylene glycol is rapidly absorbed from the GI tract and
metabolized to glycolic acid and oxalic acid. Oxalic acid is precipitated in renal tubules as oxalate
and it damage renal tubular epithelia cells. Glycolic acid is directly toxic to tubular epithelium.
Symptoms of acute renal failure (oliguria, anorexia, flank pain) are characteristic of ethylene glycol
poisoning and occur in 24 to 72 hours after the ingestion. Diagnosis can be suspected if there is high
plasma osmolality and high anion-gap metabolic acidosis. Diagnosis can be established by
measuring ethylene glycol levels in the blood.
Calcium oxalate crystals are detected by microscopy of the urine and are shaped like folded
envelopes. Although glomeruli appear normal, signs of tubular epithelial damage are clearly visible
on light microscopy. This damage manifests itself histologically as ballooning and yacuolar
degeneration, predominantly in the proximal convoluted tubules as seen on light microscopy.
(Choice C)

Choices A and E: Prolonged hypotension and septicemia produces ischemic acute tubular necrosis
(ATN). It is characterized by tubular epithelial cell desquamation, presence of granular, hyaline or
pigmented cats within tubules, interstitial edema and inflammation. Oxalate crystals will not be
seen.

Choice B: renal failure can occur with advance liver disease and is called as hepato-renal syndrome.
The hallmark of this condition is renal vasoconstriction. Kidneys are histologically normal and
resume their function following liver transplantation.
Choice D: Chronic malnutrition does not lead to tubular epithelial damage or to formation of
calcium oxalate crystals.

Educational Objective:
Ethylene glycol poisoning should be suspected in any individual presenting with acute renal failure
with evidence of anion gap metabolic acidosis and increased osmolar gap. Degree of acidosis is
disproportionate to the degree of renal insufficiency. Its ingestion leads to acute renal failure
because of precipitation of calcium oxalate crystals in renal tubules and subsequent damage to the
tubular epithelium. Typical biopsy findings include acute tubular necrosis and presence of calcium
oxalate crystals in renal tubules.
Question 43
A 6-year-old Caucasian boy is brought to your office a swollen face and complaints of passing
dusky and smoky urine. Two weeks ago, he has had a crop of pustules over his back for which he
was prescribed certain antibiotics.. A kidney biopsy is performed and the immunofluorescence
findings are shown on the slide below.

The fluorescent areas on this slide are most likely to be caused by deposition of:

A M-protein

B Complement 1q

C. Albumin

D. IgG, IgM and IgA

E. Complement 3

F Light chains

Explanation:

Taking into consideration, patient’s age, his history of past skin infection and immunofluorescent
findings on his renal biopsy, the most likely diagnosis is post-streptococcal glomerulonephritis
(PSGN). PSGN is an immunologically mediated disease usually seen in children. Patients typically
present with peri-orbital edema, cola-colored urine, oliguria, and high blood pressure. PSGN
produces a nephritic picture characterized by hematuria, azotemia, oliguria and mild to moderate
hypertension. Urinalysis will show red cell casts and mild proteinuria (2- 3 gm/day). Laboratory
studies show elevated anti-streptococcal antibody (ASO) titers and decreased serum complement
(C3) levels.
The classic light microscopy finding is that of enlarged, diffusely hypercellular glomeruli. The
hypercellularity is due to a combination of leukocyte infiltration (neutrophils and monocytes) and
mesangial and endothelial cell proliferation. Immunofluorescence microscopy shows granular
deposits of IgG and C3 in the GBM and mesangium, producing the ‘starry sky' appearance, as seen
above. The most characteristic ultrastructural features are presence of subepithelial humps,
representing deposition of antigen-antibody complexes at the epithelial surface. Most children
(595%) eventually recover with out any long term sequelae. Treatment is mainly supportive.
(Choice E)

Choice A: M- protein is a component of the streptococcal cell wall. Although streptococci are
implicated in pathogenesis of PSGN, they directly do not produce the disease and M proteins are
not present in the glomeruli.

Choice B C1q deposits may be seen in MPGN type 1 along with IgG and C3. These deposits are
subendothelial. Membranous glomerulopathy also shows C1q deposition.

Choice C: Albumin deposition is seen in diabetic nephropathy

Choice D: Deposition of all three immunoglobulins is seen in lupus nephritis and is referred to as a
‘full house’

Choice F: Light chain deposition is seen in light chain deposition disease that is characterized by
overproduction and extracellular deposition of monoclonal immunoglobulin light chains

Educational Objective:
Post-streptococcal glomerulonephritis occurs most frequently in children, but adults can also be
affected. Immunofluorescence microscopy shows granular deposits of IgG and C3 in the
mesangium and basement membranes, producing a 'starry sky’ appearance. Recovery is excellent in
the pediatric age group.
Question 44

A superficial lesion on uterine cervix appears suspicious to the clinician who performs a biopsy. A
pathologist classifies this lesion as dysplastic. Which of the following feature differentiates
dysplasia from carcinoma?

Α. Nuclear abnormalities
B. Reversibility of changes
C Cellular pleomorphism
D. Increased number of mitotic figures
E. Normal cell polarity
F. Disorderly maturation of epithelial cells

Explanation:
Dysplasia is defined as constellation of changes that include loss in uniformity of individual cells
along with loss of their architectural pattern accompanied by presence of pleomorphic cells with
hyperchromatic nuclei. Increased number of mitotic figures may be seen that are present in
abnormal location. When dysplasia is seen across the whole width of the epithelium, it is regarded
as a preinvasive neoplasm and is then known as carcinoma in situ. Thus in carcinoma in situ,
dysplasia precedes carcinoma.
In many instances, dysplasia is a reversible process that does not necessarily progress to cancer.
However this is true only for mild to moderate dysplasia. A lesion is called as high-grade dysplasia
as long as dysplastic cells do not penetrate the basement membrane. Once breach in the basement
membrane by these abnormal cells, the considered as an invasive carcinoma. Importance of
recognizing a lesion in dysplastic stage is that the disease is curative. Once it progresses to the
invasive stage, it may no longer be amenable to complete cure. (Choice B)

(Choices A, C, D, E and F) Pleomorphism, nuclear abnormalities, increased number of mitoses,


disordered maturation, and abnormal orientation of the cells are features that are common to both
dysplasia and carcinoma.

Educational Objective:

Dysplasia to a certain extent is a reversible change of the lining epithelium. Although many of its
cellular features are similar to a malignant cell, it is not regarded as an invasive cancer as long as
the dysplastic changes are confined to the epithelium. In many organs, cancer progresses through
intervening stages of dysplasia as in case of cancer of the colon..
Question 45

A 40 year old Caucasian female presents to a clinic complaining of a rash on skin of her chest wall.
Physical examination of her skin reveals induration and pitting edema over the left breast. On deep
palpation a mass is felt in her left breast. It was biopsied and the histological repost indicated
carcinoma. Which of the following is the most likely cause of the induration and edema seen in this
patient?

A. Hypoalbuminemia.
B. Lymphatic Obstruction
C. Fungal infection
D. Local anaphylactic reaction
E. Venous thrombosis

Explanation:

The skin findings in this patient are consistent with what is termed as ‘peau de orange’ or changes
resembling skin of an orange. It refers to the presence of pitting edema in subcutaneous tissue of
breast accompanied by skin thickening around exaggerated hair follicles. Most commonly it is
caused by obstruction of dermal lymphatics by neoplastic cells.
‘Peau de orange’ is classically associated with the inflammatory carcinoma of the breast, although it
can be present in other types of breast cancers. Commonly, women having inflammatory breast
cancer present with breast tenderness, reddening and widespread edema of the skin. Clinically it
simulates mastitis. Inflammatory breast cancer is a highly malignant form of breast cancer. Many
clinicians recognize this form of breast cancer as inoperable. (Choice B)

Choice A: Hypoalbuminemia produces general anasarca and not localized edema.

Choice C and D: Although patients of carcinoma of the breast may develop fungal infection and
local anaphylactic reaction, these are not directly caused due to presence of an underlying cancer.

Choice E: Venous thrombosis is more common in deep veins of the leg and pelvic vessels. ‘Peau de
orange’ is not produced due to venous thrombosis.

Educational Objective:

‘Peau de Orange’ describes the presence of pitting edema in subcutaneous breast tissue
accompanied by skin thickening around exaggerated hair follicles. The pitting edema occurs
because of widespread dissemination of neoplastic cells in dermal lymphatics (carcinomatosis of
dermal lymphatics). It is important to recognize this condition as clinically it may be misdiagnosed
as mastitis due to its ‘inflammatory’ appearance. It can be seen in a highly malignant form of breast
cancer – inflammatory carcinoma of breast.
Question 46

A 30- year- old African American male presents with cervical lymphadenopathy. His past history is
non-contributory. Lymph node biopsy reveals effaced lymph node architecture and proliferation of
lymphoid cells. Which of the following would be most consistent with a diagnosis of a lymphoid
malignancy in this case?

Answers:
A Presence of immature lymphocytes.
B. Abundant mitotic figures within the lymph node
C. Presence of Reed Sternberg cells
D. Clonal T cell receptor gene rearrangement
E. Admixture of lymphoid cells in different stage of maturation.

Explanation:
Lymphadenopathy can be produced due to inflammatory changes in the lymph node (reactive
hyperplasia), primary lymphoid malignancies or because of metastatic deposits. In majority of
instances, metastatic deposits are easy to recognize on histological examination. However,
sometime it becomes difficult to distinguish between reactive lymphoid hyperplasia and primary
malignancies of the lymphoid tissue (lymphomas and related disorders). Several patterns of reactive
hyperplasia are seen. Common patterns are follicular hyperplasia, sinus hyperplasia and diffuse
hyperplasia. These are characterized respectively, by enlargement of follicular, sinusoidal or of both
the compartments of the lymph node. Lymphomatous proliferation too can cause enlargement of
follicles (nodal lymphomas) or it may produce diffuse enlargement of lymph nodes with effacement
of architecture. In certain instances it becomes difficult to distinguish between lymphoid
hyperplasia and primary lymphoid malignancies. In such cases, ancillary studies are indicated.
Reactive lymphoid hyperplasia is polyclonal in that it consists of a proliferation of many different
cell types within the lymph node Malignant transformation in contrast is monoclonal in origin. Thus
evaluation for monoclonality of the proliferating lymphocytes becomes a useful tool for diagnosis.
The clonality of a T cell population can be assessed by molecular methods that examine the
rearrangement of T cell receptor (TCR) genes. If a single allele for the V region of the T-cell
receptor predominates in lymphocytic population monoclonal proliferation is suspected. The same
principle applies when assessing B cell lymphocytic proliferation. Clonal rearrangement of the
genes for immunoglobulin heavy chains is suggestive of a B cell lymphoma. (Choice D)

Choices A: immature lymphocytes are present in normal lymph nodes as well as in reactive
hyperplasia and lymphoma.

Choice B: Large number of mitotic figures can be seen in a reactive disorder also, since mitotic
figures represent rapid proliferation.

Choice C: Reed-Sternberg cells are seen in Hodgkin’s lymphoma as well as in some reactive
conditions.

Choice E: An admixture of several cell types in a lymph node is suggestive of a benign process. It is
the monomorphic and monoclonal nature of proliferating cells that are more indicative of a primary
neoplastic disorder of the lymphoid tissue.

Educational Objective:
Benign lymph node enlargement in response to antigenic stimulation is associated with a polyclonal
proliferation of lymphocytes. A monoclonal lymphocytic proliferation, as assessed by T cell gene
rearrangement or clonal rearrangement of immunoglobulin genes, is a strong evidence of
malignancy.
Question 47
A previously healthy 58-year-old male is brought to your office. He appears confused, lethargic and
disoriented. His family members say that patient’s personality has changed drastically within last
few weeks. He dies after 3 months despite best treatment. A section of his brain is shown below.

What is the most likely diagnosis?

Answers
A. Schwannoma
B. Oligodendroglioma
C. Fibroblastic meningioma
D. Glioblastoma multiforme
E. Primary CNS lymphoma

Explanation:

This coronal section from brain shows presence of a solitary solid parenchymal mass that is grayish
white to red in color and shows areas of hemorrhage and necrosis. Gross appearance of this mass is
compatible with a tumor. In adults, tumors commonly found in brain, represent hematogenous
metastasis from distant organs. Malignant tumors of lung, breast, kidney, colon and skin usually
metastasize to the brain. Metastasis from carcinoma of lung is the commonest secondary neoplasm
in the brain. Metastases are found more frequently in frontoparietal lobes, largely because of
maximum blood supply to this area. Metastatic tumors are usually multiple. A solitary tumor in
brain is more likely to be a primary neoplasm of CNS; although exceptions do occur. In adults most
common primary brain tumors, in order of frequency are:
1. Gilioblastoma multiforme (GBM) is an astrocytic tumor usually it is located in the cerebral
hemispheres (in the frontal or temporal lobe, or near the basal ganglia). GBM may attain large size
and may cross the midline. On macroscopic examination, GBM are white and poorly defined, with
areas of necrosis and hemorrhage. GBM are highly-malignant tumors with a very poor prognosis;
most patients die less than a year after diagnosis.
2. Meningiomas are benign tumors derived from arachnoidal cells. Usually they are well
circumscribed. They are located on the surface of the brain, where they are attached to the dura.
Meningiomas are found more often over the lateral and parasagittal brain convexities, falx cerebri,
sphenoidal ridge and olfactory groove.

3. Acoustic neuromas are a special type of schwannoma that arise from Schwann cells. Most of these
tumors are found at the cerebropontine angle.
Considering the patient’s age, solitary nature of the tumor and the rapid downhill course, the tumor
in the case is likely to be a glioblastoma multiforme (Choice D)

Choice A: Schwannoma are usually found in cerebropontine angle. These are benign slowly
growing tumors

Choice B: On grass examination oligodendroglioma present as well-circumscribed gray masses.


Calcifications may be visible; areas of necrosis and hemorrhages are not common. They are usually
slow growing tumors.

Choice C: Fibroblastic meningiomas are dura based benign tumors. They are grayish tan in color
and have a trabeculated cut surface. Necrosis is not seen in benign meningiomas, except when they
undergo infarction.

Choice E: Primary CNS lymphoma is seen in immunocompromised patients; especially those


suffering from AIDS

Educational Objective:

Gliobalstoma multiforme, meningioma and schwannoma are the most common primary brain
tumors in adults. Glioblastoma multiforme is a highly malignant tumor arising from astrocytes.
Most of the patients are dead within one year of diagnosis. On gross examination, foci of necrosis
and hemorrhage are seen on macroscopic examination.
Question 48

A 34-year-old female presents with complaints of vague abdominal pain. A mass is detected in the
liver, which on CT angiography is found to be a highly vascular tumor. Which of the following
substances is responsible for development of blood vessel in this tumor?

Answers

A. Insulin like growth factor (IGF)


B. Fibroblast growth factor (FGF)
C. Epidermal growth factor (EGF)
D. Interferon – γ (IFN-γ)
E. Interleukin 1 (IL-1)

Explanation:

Development of new blood vessels in a tumor for the purpose of sustaining its viability is called
tumor angiogenesis. In absence of angiogenesis, tumors would be unable to survive beyond 1 to 2
mm in size. Tumor blood vessels differ from vessels in normal tissue, in being able to grow
continuously. It is seen that tumor cells secrete more than a dozen factors that can stimulate
angiogenesis. Of these, two are most important: vascular endothelial growth factor (VEGF
stimulates angiogenesis in a variety of other tissues, including those that are normal, chronically
inflamed, healing, and neoplastic) and basic fibroblast growth factor (bFGF). As VEGF increases
endothelial cell motility and proliferation, new capillaries begin to sprout. bFGF is produced by a
wide range of cells, and is significantly involved in the promotion of endothelial cell proliferation,
migration and differentiation. As a group, bFGFs contribute not only to angiogenesis, but are
important in embryonic development, hemetopoiesis, and wound repair. In a developing tumor, in
addition to tumor cells, stromal cells and infiltrating inflammatory cells also secrete angiogenic
factors. Because angiogenesis is critical for growth and survival of tumors, much attention is
directed in understanding the molecular mechanisms of tumor angiogenesis. Recently some drugs
that inhibit tumor angiogenesis have been approved as a therapeutic option for treatment of cancer.
(choice B)
.
Choice A: Insulin-like growth factor (IGF) is synthesized predominantly by hepatocytes under the
influence of growth hormones. It serves to stimulate cell growth arid multiplication. IGF-1 does not
directly stimulate angiogenesis, but can indirectly promote neovascullarization by encouraging cell
growth.

Choice C: Epidermal growth factor has a mitogenic influence on epithelial cells, hepatocytes, and
fibroblasts. It does not appear to directly stimulate angiogenesis.

Choice D: Although interferon-γ can indirectly promote neovascularization through the activation of
macrophages {which can release VEGF), it does not appear to directly stimulate angiogenesis.
Choice E: Although in itself it is not angiogenic, interleukin- 1 (1L-1) because of its
proinflammatory nature, can induce release of other cytokines that trigger cellular VEGF
expression.

Educational Objective:
The key growth factors that promote angiogenesis in neoplasms and granulation tissue are vascular
endothelial growth factor (VEGF) and fibroblast growth factor (FGF). Proinflammatory cytokines
like IL-1 and INF-γ can indirectly promote angiogenesis through increased VEGF expression.
Recently some drugs that inhibit tumor angiogenesis have been approved as a therapeutic option for
treatment of cancer.
Question 49

Certain individuals who demonstrate increased activity of some specific intracellular enzyme are
more prone to develop certain chemically induced cancers. Which of the following enzymes is most
likely to be overactive in these patients?

A. Superoxide dismutase
B. Mitochondrial cytochrome oxidase
C. Glutathione-S-transferase
D. UDP- glucuronyl transferase
E. Microsomal cytochrome P-450-dependant monoxygenases

Explanation:
Many chemical exist that have the potential to transform normal cells to a malignant phenotype.
Induction of tumors because of chemicals is called chemical carcinogenesis. Hundreds of
chemicals (both natural and synthetic) are now known to induce tumors, both in vitro and in vivo.
Many of the chemicals that induce cancer exist in an inactive state; and are called procarcinogens.
Procarcinogens are metabolized in body to yield metabolites that are able to transform cells. These
metabolites are called ultimate carcinogens. In body, cytochrome P-450 dependant
monooxygenases, an enzyme system present in hepatic microsomes and endoplasmic reticulum of
many other tissues, is mainly concerned in production of ultimate carcinogens. Normally,
cytochrome P-450-dependant monooxygenases metabolize steroids, alcohol and other foreign
substances by rendering them soluble and easier to excrete. Unfortunately this metabolic process is
also concerned in converting pro-carcinogens to ultimate carcinogens, which are capable of
producing mutations.(choice E)

Choice A: Superoxide dismutase converts superoxide ions into oxygen and hydrogen peroxide
during phagocytosis. This enzyme is not involved in the metabolism al chemical carcinogens.

Choice B: Mitochondrial cytochrome oxidase is a component of the electron transport chain that
reacts with molecular oxygen to produce water. This enzyme is not involved in the metabolism of
carcinogens.

Choice C: Gluathione -S-transferase is involved in detoxification of some chemical carcinogens.


Unlike microsomal cytochrome P-450- dependant monooxygenases, this enzyme converts toxic
substances into inactive non-carcinogenic metabolites.

Choice D: UDP-glucuronyl transferase converts bilirubin into soluble bilirubin diglucuronide in the
hepatocytes. This enzyme is not involved in the metabolism of chemical carcinogens.

Educational Objective:
Most chemical carcinogens enter the body in an inactive state (i.e. as pro-carcinogens). These pro-
carcinogens are converted into active metabolites by the microsomal cytochrome -P450-dependant
monooxygenases. Individual susceptibility to chemical carcinogens depends on the activity of these
enzymes, which is genetically determined.
Question 50

A 35-year-old female presents to your office with a three months history of low-grade fever,
malaise, increasing lethargy and neck swelling Biopsy of the affected tissue reveals the following
histologic findings-

An abnormality in which of the following tissues is most likely to be responsible for this patient's
symptoms?

A. Epithelium of the skin


B. Thyroid gland
C. Lymphoid tissue
D. Myeloid precursors of blood
E. Respiratory epithelium

Explanation:

To solve the above problem, first it is important to recognize the characteristic histology. The
histologic graphic is showing that majority of cells are not forming cohesive structures (i.e. they are
lying discreetly). Most of the cells appear to be lymphocytes. In addition, neutrophils, macrophages,
reticulam cells, eosinophils and activated lymphoid cells are seen. Some binucleated giant cells are
also present. They have identical nuclei that have prominent nucleoli. All these features are
characteristic of Hodgkin’s lymphoma. Giant binucleated cells are known as Reed-Stemberg (RS)
cells. Hodgkin’s lymphoma is a primary tumor of lymph nodes (Choice C)

Choice A: Skin consists of lining keratinized stratified squamous epithelium that overlies the
dermis. Dermis consists of fibrocollgenous tissue in which various dermal appendages can be
recognized. None of these cells are present in the given illustration. Tumors arising from these
tissue shows features of the parent cells , for example metastatic squamous cell cancer should show
malignant squames.

Choice B: Hashimoto’s thyroiditis that may affect the thyroid gland may show large number of
lymphoid cells. But in addition, it will also show thyroid follicle lined by Hurthe cells. RS cells are
not a feature of Hashimoto’s thyroiditis.

Choice D: RS cells are not associated with tumor of myeloid cells i.e. myeloid leukemias. In
leukemias, precursor myeloid cells are present in different proportion depending on the type of
leukemia.

Choice E: The respiratory epithelium is a simple columnar ciliated epithelium that extends down to
the level of the bronchioles. At the bronchioles, the epithelium gradually changes into cuboidal
epithelium that transforms into flattened alveolar lining cells..

Educational Objective:

Reed-Stemberg cells are large binucleated cells that are typical of Hodgkin’s lymphoma but are not
pathognomic of this disease. They can be found in some other non- neoplastic disorders also. RS
cells when present in an appropriate mileau of lymphocytes, eosinophils, neutrophils, reticulam
cells, macrophages and activated lymphoid cells can aid in diagnosis of Hodgkin’s disease.

Vous aimerez peut-être aussi